You are on page 1of 93

Tp ch Ton hc MathVn S 02-2009

S 02 - Nm 2009
Tp ch Ton Hc dnh cho Hc sinh - Sinh vin Vit Nam
Tp ch Ton hc MathVn S 02-2009
Mc lc
Cu chuyn Ton hc
Gi thuyt Riemann Phan Thnh Nam 03
Bi vit chuyn
V p ca phn s Farey Nguyn Mnh Dng 09
Cu chuyn nh v mt nh l ln Hong Quc Khnh 20
Bt ng thc Turkevici v mt s dng m rng V Quc B Cn 39
Cc phng php tnh tch phn Nguyn Vn Vinh 48
L thuyt cc qun xe Nguyn Tun Minh 58
Cuc thi gii Ton MathVn
Ton dnh cho Hc sinh 72
Ton dnh cho Sinh vin
74
Cc vn m
74
Olympic Hc sinh Sinh vin
Olympic Sinh vin ton Belarus 2009 75
Olympic Sinh vin khoa Ton i hc Sofia 2009 76
VMO 2009 thi, li gii v bnh lun Trn Nam Dng 77
Gc Lp trnh tnh ton
th trong Mathematica 85
Tin tc Ton hc
Tin Th gii 89
Tin trong nc 92
Tp ch Ton hc MathVn S 02-2009
Cu chuyn Ton hc
Gi Thuyt Riemann
Da theo J. Brian Conrey, American Institute of Mathematics
Phan Thnh Nam, Khoa Ton - i hc Copenhagen, an Mch
Li gii thiu. Bi vit ny ca J. Brian Conrey, Director of the American Institute of Mathe-
matics, ng trn Notices of the AMS (Match 2003). Bi bo va c nhn gii thng 2008 AMS
Levi L. Conant cho cc bi vit hay nht trn cc t Notices of the AMS v Bulletin of the AMS
(http://www.ams.org/ams/press/conant-conrey-2008.html). Bi vit cho mt ci nhn tng quan
v gi thuyt Riemann, t lch s bi ton n nhng bc tin gn y. Chng ti xin lc trch na
u ca bi bo, v bn c quan tm c khuyn khch c nguyn bn bi bo ny ti a ch
http://www.ams.org/notices/200303/fea-conrey-web.pdf.
Hilbert, ti i hi Ton hc Th gii nm 1990 Paris, a Gi Thuyt Riemann vo danh
sch 23 bi ton dnh cho nhng nh Ton hc ca th k 20. By gi th n ang tip tc thch
thc nhng nh Ton hc th k 21. Gi thuyt Riemann (RHRiemann Hypothesis) tn ti
hn 140 nm, v hin ti cng cha hn l thi k hp dn nht trong lch s bi ton. Tuy nhin
nhng nm gn y chng kin mt s bng n trong nghin cu bt ngun t s kt hp gia
mt s lnh vc trong Ton hc v Vt l.
Trong 6 nm qua, Vin Ton hc M (AIMAmerican Institute of Mathematics) ti tr
cho 3 n tp trung vo RH. Ni u tin (RHI) l Seattle vo thng 8 nm 1996 ti i hc
Washington (University of Washington). Ni th hai (RHII) l Vienna vo thng 10 nm 1998 ti
Vin Schr
..
odinger (Erwin Schr
..
odinger Institute), v ni th ba (RHIII) l New York vo thng 5
nm 2002 ti Vin Ton Courant (Courant Institute of Mathematical Sciences). Mc tiu ca 3
n ny l khch l nghin cu v tho lun v mt trong nhng thch thc ln nht ca Ton
hc v xem xt nhng hng tip cn khc nhau. Liu chng ta c tin gn hn ti li gii cho
Gi thuyt Riemann sau cc n lc ? Liu c phi chng ta hc c nhiu iu v hm zeta
(zeta-function) t cc n ? iu l chc chn! Mt s thnh vin trong cc n ny ang
tip tc cng tc vi nhau trn trang web (http://www.aimath.org/WWN/rh/), ni cung cp mt
ci nhn tng quan cho ch ny.
y ti hi vng phc tho mt s hng tip cn ti RH v k nhng iu th v khi lm vic
trong lnh vc ny ti thi im hin ti. Ti bt u vi bn thn Gi thuyt Riemann. Nm 1859
trong mt bo co seminar "Ueber die Anzahl der Primzahlen unter eine gegebener Gr
..
osse", G. B.
F. Riemann ch ra mt s tnh cht gii tch cn bn ca hm zeta
(s) := 1 +
1
2
s
+
1
3
s
+ ... =

n=1
1
n
s
.
Chui ny hi t nu phn thc ca s ln hn 1. Riemann chng minh rng (s) c th m rng
bi s lin tc thnh mt hm gii tch trn c mt phng phc ngoi tr ti im s = 1 (simple
pole). Hn na ng chng minh rng (s) tha mn mt phng trnh hm th v m dng i xng
ca n l
(s) := s(s 1)

s
2

s
2

(s) = (1 s)
trong (s) l hm Gamma (Gamma-function).
Tp ch Ton hc MathVn S 02-2009
Hnh 1: (
1
2
+ it) vi 0 < t < 50
Tht ra hm zeta c nghin cu trc bi Euler v mt s ngi khc, nhng ch nh
mt hm vi bin s thc. Ni ring, Euler ch ra rng
(s) =

1 +
1
2
s
+
1
4
s
+
1
8
s
+ ...

1 +
1
3
s
+
1
9
s
+ ...

1 +
1
5
s
+ ...

1
1
p
s

1
,
trong tch v hn (gi l tch Euler) ly trn tt c cc s nguyn t. Tch ny hi t khi phn
thc ca s ln hn 1. y l mt phin bn gii tch cho nh l c bn ca s hc, rng mi s
nguyn c th phn tch mt cch duy nht thnh cc tha s nguyn t. Euler dng tch ny
chng minh rng tng nghch o ca cc s nguyn t l khng b chn. Chnh tch Euler thu
ht s quan tm ca Riemann ti hm zeta: khi ng ang c gng chng minh mt gi thuyt
ca Legendre, v trong mt dng chnh xc hn pht biu bi Gauss:
(x) := (s cc s nguyn t nh hn x)
x

2
dt
log(t)
.
Riemann to ra mt bc tin ln ti gi thuyt ca Gauss. ng nhn ra rng s phn b cc
s nguyn t ph thuc vo s phn b cc khng im ca hm zeta. Tch Euler chng t khng
c khng im no ca (s) c phn thc ln hn 1; v phng trnh hm ch ra khng c khng
im no c phn thc nh hn 0 [Ngi dch: do s i xng] ngoi cc khng im tm thng
ti s = 2, 4, 6, ... Do mi khng im phc phi nm trong di 0 Re(z) 1. Riemann a
ra mt cng thc tng minh cho (x) ph thuc vo cc khng im phc = + i ca (s).
Mt dng n gin ca cng thc ni rng
(x) :=

nx
(n) = x

log 2
1
2
log

1
1
x
2

ng nu x khng phi l ly tha ca mt s nguyn t, trong hm von Mangoldt (n) = log p


nu n = p
k
vi mt s nguyn k no v (n) = log 0 nu ngc li. Ch rng tng ny khng
hi t tuyt i (nu vy th

nx
(n) phi lin tc theo x nhng iu ny r rng khng ng).
Do phi c nhiu v hn cc khng im . y tng tnh trn vi s bi v c hiu l
lim
T

||T
. Ch rng |x

| = |x|

; do cn ch ra < 1 chng minh rng



nx
(n) x, mt
Tp ch Ton hc MathVn S 02-2009
cch pht biu khc ca gi thuyt Gauss.
Hnh 2: Biu vin Re((s)), ng Re((s)) = 0 (m), Im((s)) (chm), biu vin Im((s))
Hnh 3: Biu 3D ca |Re((s))|, ng Im((s)) (ng chm)
Phng trnh hm ta ni ban u ch ra rng cc khng im phc phi i xng vi ng
thng Re(s) =
1
2
. Riemann tnh mt s khng im phc u tin:
1
2
+ i14.134...,
1
2
+ i21.022...
v chng t rng N(T), s cc khng im vi phn o nm gia 0 v T, l
N(T) =
T
2
log
T
2e
+
7
8
+ S(T) + O(1/T)
trong S(T) =
1

arg (1/2 + iT) c tnh bi bin phn lin tc bt u t arg (2) = 0 dc


theo cc ng thng ti arg (2 +iT) = 0 ri arg (1/2 +iT) = 0. Riemann cng chng minh rng
S(T) = O(log T). Ch : ta s thy sau ny rng bc nhy gia cc khng im l 2/ log T.
Riemann cng d on rng s N
0
(T) cc khng im ca (1/2 + it) vi 0 t T l khong
T
2
log
T
2e
v sau nu ra gi thuyt rng mi khng im ca thc s u nm trn ng
thng Im(z) = 1/2; chnh l gi thuyt Riemann.
Tp ch Ton hc MathVn S 02-2009
Cc n lc ca Riemann tin gn n vic chng minh gi thuyt ca Gauss. Bc cui cng
c hon tt bi Hadamard v de la Valle Poussin, hai ngi chng minh c lp nhau trong
nm 1896 rng (s) khc khng khi phn thc ca s bng 1, v t dn ti kt lun khng nh
cho gi thuyt ca Gauss, by gi c gi l nh l s nguyn t (Prime Number Theorem).
Hnh 4: Bin i Fourier ca phn sai s trong nh l s nguyn t v

vi || < 100
Cc tng u tin
Khng my kh khn chng t RH (Riemann Hypothesis) tng ng vi khng nh rng
vi mi > 0
(x) =
x

2
dt
log t
+ O(x
1/2+
).
Tuy nhin kh khn nm ch tm ra mt cch tip cn khc vi (x) v thu cc thng tin v
cc khng im.
Mt tng ng d thy khc ca RH l khng nh M(x) = O(x
1/2+
) vi mi > 0, trong

M(x) =

nx
(n)
v (n) l hm Mobius c nh ngha t chui Dirichlet sinh 1/
1
(s)
=

n=0
(n)
n
s
=

1
1
p
s

.
Vy nu p
1
, ..., p
k
l cc s nguyn t phn bit th (p
1
...p
k
) = (1)
k
; v (n) = 0 nu n chia
ht cho p
2
vi mt s nguyn t p no . Chui ny hi t tuyt i khi Re(s) > 1. Nu c lng
M(x) = O(x
1/2+
) ng vi mi > 0 th bng cch ly cc tng ring phn ta thy chui hi t
vi mi s c phn thc ln hn 1/2; ni ring khng c khng im no ca (s) nm trn na mt
phng m ny, bi v khng im ca (s) l im k d (poles) ca 1/(s) [Ngi dch: v do tnh
i xng nn cng dn n khng c khng im no nm trn na mt phng m Re(s) < 1/2, v
do mi khng im u ch nm trn ng thng Re(s) = 1/2]. Ngc li, RH suy ra c lng
ny cho M(x), iu ny cng khng kh chng minh.
Thay v phn tch trc tip (s), c v s d dng hn khi lm vic vi M(x) v chng minh
c lng trn. Tht ra, Stieltjes thng bo rng ng c mt chng minh nh vy. Hadamard,
trong chng minh ni ting nm 1896 v Prime Number Theorem, dn ra tuyn b ca Stieltjes.
Tp ch Ton hc MathVn S 02-2009
Hnh 5: 1/|(x + iy) vi 0 < x < 1 v 16502.4 < y < 16505
Hadarmard ni rng nh l ca ng yu hn nhiu, v ch chng minh (s) khc 0 trn ng thng
Re(s) = 1, nhng hi vng tnh n gin ca chng minh s c ch. Stieltjes, tuy nhin, sau khng
bao gi cng b chng minh ca mnh.
Mertens d on mt gi thuyt mnh hn rng
M(x)

x,
iu r rng dn n RH. Tuy nhin gi thuyt ca Mertens b chng minh l sai bi Odlyzko
v te Riele nm 1985. c lng M(x) = O(

x) thm ch dng RH nh mt l chn: ng tng gi


bu thip ti ng nghip Harald Bohr trc khi qua English Channel trong mt m bo t, tuyn
b l ng chng minh xong RH. Thm ch Hardy l mt ngi v thn, ng cng tin mt cch
tng i v Cha, rng nu Cha tn ti, cng chng thnh tu ti trong mt hon cnh nh vy!
Hilbert c v hi mu thun khi nhn nhn v kh ca RH. Mt ln ng so snh ba bi ton
m: tnh siu vit ca 2

2
, nh l ln Fermat, v gi thuyt Riemann. Theo quan im ca ng,
RH c th s c gii trong vi nm, nh l ln Fermat c th c gii khi ng cn sng, v cu
hi v s siu vit c th s khng bao gi c tr li. ng ngc nhin l cu hi v s siu vit
c gii trong vi nm sau bi Gelfond v Schneider, v, d nhin, Andrew Wiles gn y
chng minh nh l ln Fermat [Ngi dch: vy nu o ngc d on ca Hilbert th c th RH
s khng bao gi c gii]. Tuy nhin trong mt dp khc Hilbert li ni rng nu ng ta sng li
sau mt gic ng 500 nm th cu hi u tin s l: RH c c gii hay cha.
Khi gn kt thc s nghip, Hans Rademacher, ngi c bit bi cng thc chnh xc cho s
cc cch phn hoch mt s nguyn, ngh rng ng c mt phn chng minh cho RH. Siegel
kim tra kt qu ny, cng vic da trn kt lun rng mt hm nht nh s c mt ni rng
gii tch bi lin tc nu RH ng. Cng ng Ton hc c gng lm cho Tp ch Time (Time
magazine) quan tm cu chuyn. Time thch th v ng mt bi bo sau khi ngi ta tm ra
li sai trong chng minh ca Rademacher.
Tp ch Ton hc MathVn S 02-2009
Cc chng c ca gi thuyt Riemann
Hnh 6: Cng thc chnh xc ca (x) s dng 100 cp khng im u tin
Sau y l mt s l do tin vo RH.
Hng t khng im khng th sai. Gn y, van de Lune ch ra 10 t khng im u
tin nm trn ng thng Re(s) = 1/2. Ngoi ra, mt d n vi s chung sc nhiu my tnh
t chc bi Sebastian Wedeniwski, chng trnh c nhiu ngi hng ng, khng nh
rng h kim tra 100 t khng im u tin nm trn ng thng . Andrew Odlyzko tnh
hng triu khng im gn cc khng im th 10
20
, 10
21
v 10
22
(c th xem trn website ca ng).
Hu ht tt c cc khng im u nm rt gn ng thng Re(s) = 1/2. Tht s ngi ta
chng minh rng c hn 99 phn trm cc khng im = + i tha mn | 1/2| 8/ log().
Ngi ta chng minh c rt nhiu khng im nm trn ng thng Re(s) = 1/2. Selberg
t c mt t l dng, v N. Levinson ch ra t nht l 1/3; t l ny sau c ci thin ln
40 phn trm. Ngoi ra RH cng ng rng mi khng im ca mi o hm ca (s) nm trn
ng thng Re(s) = 1/2. Ngi ta chng minh c rng c nhiu hn 99 phn trm cc khng
im ca o hm bc ba

(s) nm trn ng thng Re(s) = 1/2. Lc gn cui i Levinson


ngh rng ng c mt phng php cho php o ngc nh l Rolle trong trng hp ny, tc l
nu

(s) c t nht mt t l dng cc khng im nm trn ng thng th iu ny cng


ng vi (s), v tng t vi

(s),

(s) ... Tuy nhin cha ai c th hin thc ha tng ca ng.


Phng php thng k. Vi t hu ht cc dy ngu nhin gm 1 v +1, hm tng tng ng
ca x b chn bi x
1/2+
. Dy Mobius c v kh ngu nhin.
S i xng ca cc s nguyn t. RH ni rng cc s nguyn t phn b theo cch p nht
c th. Nu RH sai th s c nhng iu bt thng trong s phn b cc s nguyn t; khng im
u tin c phn thc khc 1/2 chc chn s l mt hng s ton hc rt quan trng. Tuy nhin,
c v t nhin khng khc nghit ti nh vy!
Tp ch Ton hc MathVn S 02-2009
V p ca phn s Farey
Nguyn Mnh Dng, Hc sinh lp 12A2 Ton, Trng HKHTN-HQG H Ni
A - M u
Trong lch s ca ton hc, nhiu khi nhng li gii, nhng nh l mi c tm ra bi nhng
ngi nghip d, nhng ngi lnh vc khc. Chnh iu ny gp phn lm cho cc kha cnh
ca ton hc a dng hn, th v hn. Trong bi vit ny, ti xin c trao i vi cc bn v phn
s Farey, gn lin vi tn tui ca nh a l hc John Farey (1766-1826) khi ng cng b nhng
tnh cht th v ca phn s Farey trn mt tp ch Trit hc di dng phng on.
nh ngha. Tp hp F
n
cc phn s Farey bc n, gi l chui Farey bc n, l tp hp ca cc
phn s ti gin thuc khong [0, 1] vi mu s khng vt qu n v c sp xp theo th t tng
dn. Do
h
k
thuc F
n
nu
0 h k n, (h, k) = 1
Cc s 0, 1 gi l cc phn t c s ca mi tp hp phn s Farey v vit c di dng
0
1
v
1
1
.
Ta c th biu din phn s Farey nh sau:
Hoc di dng cy Stern:
Tp ch Ton hc MathVn S 02-2009
B - Tnh cht
Chng ta hy cng xt cc tnh cht th v ca phn s Farey
nh l 1. Nu
h
k
v
h

l hai phn t lin tip ca F


n
th
(k +k

) > n
Chng minh
Xt phn s
h+h

k+k

(phn s ny c gi l trung bnh ca


h
k
v
h

). Khi
h +h

k +k


h
k
=
kh

hk

k(k +k

)
> 0
V
h


h +h

k +k

=
kh

hk

(k +k

)
> 0
Do
h +h

k +k


_
h
k
,
h

_
Nn nu k +k

n th
h +h

k +k

F
n
. iu ny l v l v
h
k
v
h

l hai phn t lin tip. nh


l c chng minh. Chng ta s quay li tnh cht ny phn sau.
nh l 2. Khng c hai phn t lin tip no ca F
n
c mu s ging nhau.
Chng minh
Nu k > 1 v
h
k
,
h

k
l hai phn t lin tip trong F
n
, khi h + 1 h

< k. Mt khc
h
k
<
h
k 1
<
h + 1
k

h

k
Do d
h
k 1
l mt phn t nm gia 2 phn t lin tip
h
k
,
h

k
, v l. Ta c iu phi chng
minh.
nh l 3. Nu
h
k
v
h

l hai phn t lin tip ca F


n
th
hh

kk

= 1
Chng minh
u tin ta cn chng minh mt b
B 1. Nu (h, k) l cc s nguyn dng nguyn t cng nhau th khi tn ti cc s nguyn
dng (x, y) sao cho
kx hy = 1
Chng minh.
Xt cc s nguyn
k, 2k, 3k, , (h 1)k
Tp ch Ton hc MathVn S 02-2009
v s d ca chng khi chia cho h. Cc s d ny u khc nhau. Tht vy, nu
k
1
k = q
1
h +r, k
2
k = q
2
h +r
vi k
1
, k
2
{1, 2, , (h 1)} th
(k
1
k
2
)k = (q
1
q
2
)h 0 (mod h)
M k
1
, k
2
{1, 2, , (h 1)} nn k
1
k
2
< h. Do k
1
k
2
= 0.
D thy rng kk

0 (mod h) vi mi k {1, 2, , (h1)}. Do t nht mt s trong cc s


1.k, 2.k, , (h 1)k c s d l 1 khi chia cho h, suy ra tn ti x {1, 2, , (h 1)} v y Z
+
.
Quay li nh l cn chng minh, nu (x
0
, y
0
) l mt nghim ca phng trnh trn, khi
(x
0
+rh, y
0
+rh) cng l mt nghim vi mi s nguyn r. Chng ta c th chn r sao cho
n k < y
0
+rk n
t x = x
0
+rk, y = y
0
+rk, khi (x, y) l mt nghim ca phng trnh trn v tha mn
(x, y) = 1, 0 n k < y n
Do
x
y
ti gin v y n nn
x
y
l mt phn t ca F
n
. Ta cng c
x
y
=
h
k
+
1
ky
>
h
k
Suy ra
x
y
nm sau
h
k
trong F
n
. Nu
x
y
=
h

th
x
y
cng nm sau
h
k
, khi
x
y

h

=
k

x h

y
ky


1
k

m
h


h
k
=
kh

hk

kk


1
kk

V vy
1
ky
=
kx hy
ky
=
x
y

h
k

1
k

y
+
1
kk

=
k +y
kk

y
>
n
kk

y

1
ky
(theo nh l 1)
V l, vy
x
y
=
h

do kh

hk

= 1.
nh l 4. Nu
h
k
,
h

v
h

l ba phn t lin tip ca F


n
th
h

=
h +h

k +k

Chng minh.
T nh l 3 ta thu c
kh

hk

= 1, k

= 1 (3.1)
Tp ch Ton hc MathVn S 02-2009
Gii h phng trnh trn theo n h

v k

ta c
h

=
h +h

kh

hk

, k

=
k +k

kh

hk

Hay
h

=
h +h

k +k

y chnh l nh l 4.
Nhn xt. Ch rng nh l 3 v nh l 4 l tng ng, ta c th suy ra nh l 3 t
nh l 4 bng php quy np nh sau:
Gi s rng nh l 4 ng vi mi F
n
v nh l 3 ng ti F
n1
, ta s chng minh n cng
ng vi F
n
. Hin nhin rng n tng ng vi vic chng minh phng trnh (3.1) tha mn
khi
h

thuc F
n
nhng khng thuc F
n1
, do k

= n. Trong trng hp ny, theo nh l 4,


k, k

< k

v
h
k
,
h

l hai phn t lin tip trong F


n1
. T nh l 4 v gi thit
h

ti gin, ta t
h +h

= h

, k +k

= k

vi l mt s nguyn. Do k, k

< k

, nn = 1. Do
h

= h +h

, k

= k +k

kh

hk

= kh

hk

= 1
Tng t
k

= 1
Php chng minh ny gi cho ta mt li gii khc cho nh l 3:
nh l ng vi n = 1, gi s n ng ti F
n1
ta cn chng minh minh n cng ng vi F
n
.
Gi s
h
k
v
h

l hai phn t lin tip trong F


n1
nhng b chia ra trong F
n
bi
h

. t
kh

hk

= r > 0, k

= s > 0
Gii h phng trnh ny theo n h

, k

vi iu kin kh

hk

= 1 ta thu c
h

= sh +rh

, k

= sk +rk

T kt hp vi (h

, k

) = 1 nn (r, s) = 1. Xt tp hp S bao gm tt c cc phn s c dng


p
q
=
h +h

k +k

vi , l cc s nguyn dng c (, ) = 1. Do
h

S, Mi phn t ca tp hp S u nm
gia
h
k
v
h

do mi c chung ca p v q u chia ht cho


k(h +h

) h(k +k

) = , h

(h +h

) k

(k +k

) =
Do mi phn t ca S u l phn t ca mt s chui Farey no , khi phn s u
tin xut hin phi c q nh nht. Suy ra = = 1, Vy phn s ny phi l
h

. Nn
h

= h +h

, k

= k +k

nh l c chng minh.
Tp ch Ton hc MathVn S 02-2009
Hai php chng minh trn cho nh l 3 khng phi l duy nht, cc bn c th tham kho
cch chng minh bng hnh hc kh hay ca G.H.Hardy hoc dng nh l Pick, chi tit cc bn c
th tham kho [1].
nh l 5. Tng ca cc t s bng mt na tng cc mu s trong chui Farey bc n.
Chng minh
u tin ta chng minh mt b .
B 2. Nu
h
k
l mt phn t ca chui F
n
th
k h
k
cng l mt phn t ca chui.
Chng minh
Do (h, k) = 1 v 0
h
k
1 nn (k h, k) = 1 v 0 1
h
k
1. Ta c pcm.
Quay li bi ton, k hiu

l tng ca tt c cc phn t ca chui F
n
.
p dng B 2, ta c

h =

(k h) nn 2

h =

k. y l chnh l kt qu ca nh
l 5.
nh l 6. Tng ca cc phn t ca chui Farey F
n
bng
n
2
Chng minh
Theo B 2, ta c

h
k
=
_
1
h
k
_
nn
2

h
k
=

1 = n
y l iu phi chng minh.
nh l 7. Trong chui Farey bc n F
n
, mu s ca phn s lin trc v lin sau phn s
1
2
bng s nguyn l ln nht khng vt qu n.
Chng minh
Gi
h
k
l phn s lin trc
1
2
trong F
n
, khi k 2h = 1 nn k l. Theo nh l 3, ta c
k + 2 > n nn k n 1, m k n nn k l s nguyn l ln nht khng vt qu n.
C - M rng
Phn s Farey v Phi hm Euler
Trong phn trc, ta lm quen vi mt s tnh cht c bn ca phn s Farey, vn t ra
y l c bao nhiu phn s Farey trong mt chui Farey bc n? Chng ta xut pht t mt nhn
xt n gin: Do tt c cc phn s Farey u dng ti gin, nn vi mt mu s b cho trc, s
t s nh hn b v nguyn t cng nhau vi b l (b), gi l Phi-hm Euler. (Chi tit v cc tnh
cht cng nh cc php chng minh ca Phi-hm Euler, cc bn c th tham kho [3].) Ta c th
s dng tnh cht ny cho mi nguyn t 2 n n. Do ta c th tnh c s phn s c trong
F
n
(k c hai phn s c s
0
1
v
1
1
) l
N = 2 +(2) +(3) + +(n)
Tp ch Ton hc MathVn S 02-2009
Ta c th tnh c (n) vi n > 1 qua cng thc
(n) = n

p|n
_
1
1
p
_
V d khi n = 7 ta c
N = 2 +(2) +(3) + +(7) = 2 + 1 + 2 + 2 + 4 + 2 + 6 = 19
ng vi kt qu trong bng phn nh ngha. V d ta c th tnh khi n = 100 th N = 3045.
Do (x) lun l s chn ngoi tr trng hp x = 1, 2 nn ta c
N = 2 +(2) +(3) + +(n)
lun l mt s l. Do s phn s cch u
1
2
lun bng nhau. y l mt cch chng minh khc
cho B 2.
Vi n rt ln th vic tnh N tr nn kh khn hn rt nhiu. Nhng nh mt tnh cht ca Phi
hm Euler ta c th tnh ton d dng hn:
(1) +(2) + +(n)
3n
2

2
Do
N 1 +
3n
2

2
Gi tr xp x ny s ngy cng chnh xc hn khi gi tr ca n tng. V d vi n = 100, theo
cng thc trn ta tnh c N = 1 +
3.100
2

2
3040, 635... trong khi gi tr chnh xc ca N l 3045.
Ta c th lp bng tnh nh sau:
S PHN T CA CHUI FAREY
n (n) N = 1 +

(n) 1 + 3n
2
/
2
1 1 2 1,30
2 1 3 2,22
3 2 5 3,74
4 2 7 5,86
5 4 11 8,60
6 2 13 11,94
7 6 19 15,90
8 4 23 20.46
9 6 29 25,62
10 4 33 30,40
15 8 73 69,39
25 20 201 190,98
50 20 775 760,91
100 40 3045 3040,63
200 80 12233 12159,54
300 80 27399 27357,72
400 160 48679 48635,17
500 200 76115 75991,89
Tp ch Ton hc MathVn S 02-2009
Phn s Farey v ng trn Ford
Mt trong nhng m rng lin quan n hnh hc ca phn s Farey l ng trn Ford.
nh ngha. Xt h trc ta Oxy. Vi mi phn s ti gin
p
q
nm trn trc Ox, ta dng
cc ng trn tip xc vi Ox ti im , tm c ta d l
_
p
q
,
1
2q
2
_
, c gi l ng trn Ford,
k hiu C(p, q).
Mt s hnh nh p ca ng trn Ford sau khi c cch iu
Tp ch Ton hc MathVn S 02-2009
Ta c mt rt c bn sau ca ng trn Ford:
Tnh cht 1. Hai ng trn Ford C(h, k) v C(h

, k

) hoc tip xc vi nhau, hoc nm ngoi


nhau. iu kin hai ng trn ny tip xc l |hk

k| = 1.
Chng minh
Gi D l khong cch gia tm ca 2 ng trn. r, R tng ng l bn knh ca ng trn
C(h, k), C(h

, k

).
Khi ta c
(r +R)
2
=
_
1
2k
2
+
1
2k
2
_
2
Xt hiu s D
2
(r +R)
2
:
D
2
(r +R)
2
=
_
h
k

h

_
2
+
_
1
2k
2

1
2k
2
_
2

_
1
2k
2
+
1
2k
2
_
2
=
(hk

k)
2
1
k
2
k
2
0
Du ng thc xy ra khi v ch khi |hk

k| = 1.
T tnh cht ny ta d dng thy c bt c hai phn s Farey lin tip no c biu din
trn h trc ta cng c hai ng trn Ford tip xc vi nhau. Ta c mt v d minh ha sau
vi chui Farey bc 7.
Tnh cht 2. Gi s
h
k
<
h

<
h

l ba phn t lin tip ca F


n
. Khi C(h, k) v C(h

, k

)
tip xc vi nhau ti im
A
1
=
_
h


k
k

(k
2
+k
2
)
,
1
k
2
+k
2
_
Tp ch Ton hc MathVn S 02-2009
v C(h

, k

) v C(h

, k

) tip xc vi nhau ti im
A
2
=
_
h

+
k

(k
2
+k
2
)
,
1
k
2
+k
2
_
Chng minh
K hiu di cc ng nh trong hnh v.
p dng nh l Thales ta c
a
h
k

h

=
1
2k
2
1
2k
2

1
2k
2
=
b
1
2k
2

1
2k
2
Do
a =
k
k

(k
2
+k
2
)
, b =
k
2
k
2
2k
2
(k
2
+k
2
)
Ta im A
1
= (x
1
, y
1
), trong
x
1
=
h

a =
h


k
k

(k
2
+k
2
)
, y
1
=
h

+
1
2k
2

1
2k
2
b =
1
k
2
+k
2
Tng t ta cng tnh c to ca A
2
.
Kt thc bi vit, ti xin nu ra mt s bi tp cc bn luyn tp thm.
Bi tp 1. Hai phn s
a
b
v
c
d
c gi l ng bc nu (c a)(d b) 0. Chng minh rng
bt k hai phn t lin tip no ca chui Farey bc n cng ng bc.
Bi tp 2. Cho a, b, c, d l cc s nguyn dng tha mn
a
b
<
c
d
v , l cc s nguyn dng.
Chng minh rng
=
a +c
b +d
nm gia hai phn s
a
b
,
c
d
v (c d)(b a) =

. Khi = , chnh l trung bnh ca


a
b
,
c
d
.
Bi tp 3. (Hurwitz) Cho mt s v t , khi tn ti v s phn s hu t
a
b
sao cho
_

a
b
_
<
1

5a
2
Hn na ta khng th thay th
1

5
bng mt s nh hn.
Tp ch Ton hc MathVn S 02-2009
Ti liu tham kho
[1] G. H. Hardy and E. M. Wright, An Introduction to the Theory of Numbers, Fifth Edition,
Oxford Science Publications, 1996.
[2] J. H. Conway and R. K. Guy, The Book of Numbers, Springer-Verlag, NY, 1996.
[3] David M, Burton, Elementary number theory, 6th Edition, Mc Graw Hill, 2007.
[4] A. H. Beiler, Recreations in the Theory of Numbers, Dover, 1966
[5] Apostol, T. M. , Modular Functions and Dirichlet Series in Number Theory, 2nd ed. New
York: Springer-Verlag, 1997.
[6] Ford, L. R, Fractions, Amer. Math. Monthly 45, 586-601, 1938.
[7] Weisstein, Eric W, Ford Circle, From MathWorldA Wolfram Web Resource.
http://mathworld.wolfram.com/FordCircle.html
Tp ch Ton hc MathVn S 02-2009
Bi vit Chuyn MathVn
Cu chuyn nh v mt nh l ln
Hong Quc Khnh - Lp 12A10 THPT Chuyn Vnh Phc, Tnh Vnh Phc
A - S lc v nh l Pascal v php chng minh
Mt nh l c Descartes khng nh l bao hm c c bn cun sch u ca Apolonius,
tt nhin l mt nh l ln, chnh l nh l Pascal. nh l Pascal chc khng cn qu xa l vi
nhng bn yu ton v c bit l yu thch hnh hc v bi vit ny ch l mt tm ti nh ca
tc gi cp n nhng ng dng ca nh l tuyt m y trong ton ph thng. nh l Pascal
tng qut c pht biu cho cc ng cnic trong mt phng x nh nhng y chng ta s
ch cp n mt trng hp c bit ca n, l vi ng trn trong mt phng, c th nh sau:
nh l. Cho su im bt k A, B, C, A

, B

, C

cng thuc mt ng trn (O). Khi giao


im nu c ca tng cp ng thng (AB

, A

B), (BC

, B

C), (CA

, C

A) s thng hng.
Chng minh (Jan van Yzeren)
y l mt nh l p v cng c rt nhiu chng minh p cho n (Cc bn c th xem thm
[1], [2], [3], [4]) y tc gi s ch trnh by mt chng minh kh th v v t quen bit, chng minh
ny c s dng mt b sau:
B . Cho hai ng trn phn bit ct nhau A v B. Hai im C, E thuc ng trn th
nht, hai im D, F thuc ng trn th hai sao cho C, A, D thng hng; E, B, F thng hng. Th
th: CE//DF.
Chng minh b
Nhn thy: (CE, CA) (BE, BA) (BF, BA) (DF, DA) (mod)
Suy ra CE//DF.
Tr li chng minh nh l:
Tp ch Ton hc MathVn S 02-2009
Gi giao im ca tng cp ng thng (AB

, A

B), (BC

, B

C), (CA

, C

A) ln lt l M, N, P.
Gi (O

) l ng trn i qua C, P, C

. B

C v BC

ct li (O

) tng ng Q, T.
S dng b trn ta c: AB

//QP nn MB

//QP (1); BB

//TQ (2); BA

//PT nn BM//PT
(3).
T (1), (2) v (3) suy ra tn ti mt php v t bin tam gic BMB

thnh tam gic TPQ


Do : BT, MP, B

Q ng quy ti tm v t y. Ni cch khc BC

, B

C v MP ng quy. T
y suy ra iu cn chng minh.
B - Mt s ng dng ca nh l Pascal trong hnh hc s cp
I. ng dng ca nh l Pascal vi su im phn bit
Chng ta cng bt u vi mt bi ton kh quen thuc:
Bi ton 1. Cho tam gic ABC ni tip ng trn (O). Gi A

, B

, C

ln lt l cc im
chnh gia ca cc cung BC, CA, AB khng cha A, B, C ca (O). Cc cnh BC, CA, AB ct cc
cp on thng C

v A

; A

v B

; B

v C

ln lt cc cp im M v N; P v
Q; R v S. Chng minh rng MQ, NR, PS ng quy.
Li gii
Tp ch Ton hc MathVn S 02-2009
Gi I l tm ng trn ni tip ca tam gic ABC.
S dng nh l Pascal cho su im A, B, C, A

, B

, C

ta thu c S, I, P thng hng (1)


Hon ton tng t: M, I, Q thng hng (2); N, I, R thng hng (3)
T (1), (2) v (3) suy ra MQ, NR, PS ng quy I.
Bi ton 2. Cho tam gic ABC ni tip ng trn tm O. Gi M l im no trn cnh
AC (M khc A, C). ng thng BM ct ng trn ln na ti N. ng thng qua A vung gc
vi AB v ng thng qua N vung gc vi NC ct nhau ti im Q. Chng minh rng QM lun
i qua mt im c nh khi M di chuyn trn cnh AC. (Bi T4/294 Tp ch Ton hc v Tui tr)
Li gii
K cc ng knh BD, CE ca (O).
Theo gi thit bi ton ta thy ngay: E, N, Q thng hng; A, D, Q thng hng.
By gi p dng nh l Pascal cho su im A, B, C, N, D, E ta suy ra O, M, Q thng hng, do
QM lun i quan mt im c nh chnh l O.
Tip n l mt nh l rt p v ni ting ca hnh hoc - nh l Lyness cng cch chng minh
rt th v bng nh l Pascal:
Bi ton 3. Cho tam gic ABC ni tip (O), ngoi tip (I). Mt ng trn (O

) tip xc trong
vi (O) v tip xc vi hai cnh AB, AC ln lt ti S, M, N. Chng minh rng I thuc MN.
Li gii
Trc tin cn chng minh mt b :
B . Cho ng trn (O) vi dy cung AB. Mt ng trn (I) tip xc trong vi (O) v tip
xc vi AB ln lt ti M,N. Th th MN i qua im chnh gia cung AB khng cha M ca (O).
Tp ch Ton hc MathVn S 02-2009
Chng minh b
MN ct li (O) P.
rng: (MN, MI) (MP, MO) (mod)
Ch tam gic IMN v OMP cn I, O ta s suy ra: (IM, IN) (OM, OP) (mod)
Suy ra OP//IN, m IN vung gc vi AB nn OP cng vung gc vi AB, suy ra iu cn
chng minh.
Tr li bi ton:
S dng b ta d thy: SM, CI v (O) ng quy ti mt im F; SN, BI, (O) ng quy ti
mt im E.
By gi s dng nh l Pascal cho su im A, B, C, S, E, F ta suy ra M, I, N thng hng.
Bi ton 4. Cho tam gic ABC ni tip ng trn (O) v mt im S trong mt phng.
AS, BS, CS ct li (O) tng ng D, E, F. Mt ng thng d qua S ct BC, CA, AB ln lt ti
M, N, P. Chng minh rng: DM, EN, FP v ng trn (O) ng quy.
Li gii
Tp ch Ton hc MathVn S 02-2009
Gi giao im th hai ca DM v (O) l V , V E ct AC N

.
S dng nh l Pascal cho b su im V, A, E, C, D, B ta suy ra M, S v N

thng hng.
T ta khng nh c N

chnh l N,nn DM, EN v (O) ng quy (1).


Tng t c DM, FP, (O) ng quy (2). T (1) v (2) suy ra iu cn chng minh.
y l bi ton kh c trng trong vic vn dng nh l Pascal, s n gin ca li gii trnh
by trn kh c th gp c trong mt phng n khc.
By gi chng ta s n vi mt bi ton ng quy th v tng gp mt trong k thi Olympic
ton Quc t nm 1996, vi bi ton ny c kh nhiu li gii p cho n v li gii s dng nh
l Pascal cng l mt trong s :
Bi ton 5. Cho P l mt im nm trong tam gic ABC sao cho APBC = APCB.
Ly D, E tng ng l tm ng trn ni tip ca cc tam gic APB v APC. Chng minh rng
AP, BD v CE ng quy.
Li gii
Tp ch Ton hc MathVn S 02-2009
Gi (O) l ng trn ngoi tip tam gic ABC; AP, BP, CP ct ng trn (O) ln th hai
S, M, N tng ng; BD, CE ct nhau I v ct ng trn (O) ln th hai tng ng T, V .
Th th MV, MT chnh l cc ng phn gic ca tam gic AMN (1)
Ta thy: SAM = APB = PBS = APB ASB = APB C
= APC B = APC ASC = NCS = SAN
Do vy AS l ng phn gic ca tam gic AMN (2)
T (1) v (2) suy ra AS, NT, MV ng quy ti mt im Q.
S dng nh l Pascal cho su im B, N, V, T, M, C ta suy ra Q, I, P thng hng.
T y d thy iu cn chng minh.
Bi ton sau l mt kt qu ng quy rt p ca Darij Grinberg, thot nhn bi ton c v
rc ri nhng nu phn tch ngc bng nh l Pascal th mi th li tr nn tht r rng v n gin.
Bi ton 6. Cho tam gic ABC ni tip ng trn (O) v ba im M, N, P cng thuc ng
thng d. AM, BM, CM ct li (O) tng ng A
1
, B
1
, C
1
; A
1
N, B
1
N, C
1
N ct li (O) tng ng ti
A
2
, B
2
, C
2
; A
2
P, B
2
P, C
2
P ct li (O) ln lt ti A
3
, B
3
, C
3
. Chng minh rng AA
3
, BB
3
, CC
3
, d
ng quy.
Li gii
Gi s AA
3
, BB
3
ct nhau ti S, giao im ca B
1
A
3
, B
3
A
1
l V .
Tp ch Ton hc MathVn S 02-2009
S dng nh l Pascal cho su im B
1
, A
3
, B
3
, A
1
, A
2
, B
2
ta suy ra N, P, V thng hng hay V
nm trn d (1)
Li p dng nh l Pascal cho su im B
1
, A, B, A
1
, A
3
, B
3
suy ra M, S, V thng hng, kt hp
vi (1) ta c S thuc d (2)
Tng t trn, nu gi giao im ca l S th S nm trn d (3)
T (2) v (3) ta s c iu cn chng minh.
Bi ton 7 di y l mt kt qu rt p v hai im ng gic v i km n chnh l mt li
gii cng rt p v kho lo bng nh l Pascal, mi cc bn cng "thng thc":
Bi ton 7. Cho P v Q l hai im lin hp ng gic i vi tam gic ABC.T P k
PP
1
BC, PP
2
CA, PP
3
AB. T Q k QQ
1
BC, QQ
2
CA, QQ
3
AB. Gi giao im
ca cc cp ng thng (P
2
Q
3
, P
3
Q
2
), (P
1
Q
3
, P
3
Q
1
), (P
1
Q
2
, P
2
Q
1
) ln lt l X
1
, X
2
, X
3
. Chng
minh rng X
1
, X
2
, X
3
, P, Q thng hng.
Li gii
Gi O l trung im ca PQ, ta bit rng cc im P
1
, P
2
, Q
1
, Q
2
, Q
3
cng thuc mt ng
trn tm O.
By gi, k cc ng knh Q
1
T, Q
3
V ca (O).
Ta thy ngay: T, P, P
1
thng hng, v V, P, P
3
thng hng.
Tp ch Ton hc MathVn S 02-2009
Do vy, s dng nh l Pascal cho b su im P
3
, Q
3
, T, P
1
, Q
1
, V ta thu c O, P, X
2
thng
hng hay X
2
thuc PQ.
Hon ton tng t ta cng c X
1
, X
3
cng thuc PQ.
Kt thc mc mt ny s l mt bi ton n gin m tc gi tin tng rng ai ang c bi
vit ny cng gii c n, tuy nhin li gii sau y bng nh l Pascal ca Greg tht s th v...
Bi ton 8. Cho hai im C, D nm trn ng trn (O) ng knh AB. AD ct BC I. K
IH vung gc vi AB. Chng minh rng IHC = IHD.
Li gii
Ly P, Q i xng vi C, D qua AB. Ta d thy P, Q thuc (O).
Gi giao im ca BP v AQ l J, ca PD v CQ l H

.
T tnh cht ca php i xng trc ta c H

nm trn AB, IJ vung gc vi AB


S dng nh l Pascal cho su im A, B, C, D, P, Q ta c I, H

, J thng hng, kt hp vi
trn ta c H

chnh l H.
Nh vy IHC = PHJ = IHD.
II. p dng nh l Pascal cho su im khng phn bit
Nh cc bn thy phn trc, nh l Pascal lun c s dng vi su im hon ton phn
bit, tuy nhin nh l Pascal vn ng nu nh su im y c th khng phn bit (Cc bn c
th chng minh tng t nh khi chng minh vi su im phn bit), l mt iu th v v
mc ny chng ta s nghin cu ng dng ca nh l Pascal trong nhng trng hp . Ta c bi
ton m u:
Bi ton 9. Cho hnh ch nht ABCD ni tip (O). Tip tuyn ca (O) ti A ct CD S.
BS ct li ng trn T. Chng minh rng CT, SO v AD ng quy.
Li gii
Tp ch Ton hc MathVn S 02-2009
Gi giao im ca CT v AD l I. S dng nh l Pascal cho su im A,B,C,D,T,A ta suy ra
S,I,O thng hng. Do nhn c iu cn chng minh.
Tip theo chng ta s cng xem xt mt bi ton chng minh ng thng i qua im c nh
ca tc gi bi vit ny, n c mt trong [6a] nhng y chng ta s khng trnh by phng
n cc v i cc cho n m s trnh by mt li gii rt ngn gn s dng nh l Pascal.
Bi ton 10. Cho tam gic cn ABC (AB = AC) ni tip ng trn (O). K ng knh AD
ca ng trn. S l mt im di ng trn ng trn. SB ct AC M, SD ct BC N. Chng
minh rng MN lun i qua mt im c nh.
Li gii
Tp ch Ton hc MathVn S 02-2009
Gi s BM, AN ct li (O) tng ng S, I; tip tuyn ca (O) ti C ct SI T
Ch rng SN, IN tng ng l phn gic ca BSC, BIC.
Do
SB
SC
=
NB
NC
=
IB
IC
V vy BSCI l t gic iu ha; nn SI, tip tuyn ti B v tip tuyn ti C ca (O) ng
quy, ni cch khc T chnh l giao im ca hai tip tuyn ti B, C ca (O) nn T c nh.
Cui cng s dng nh l Pascal cho su im A, B, C, C, S, I suy ra M, N, T thng hng v
nhn c iu cn chng minh.
Bi ton 11. Cho tam gic ABC v im S thuc cnh BC. Trn cc tia AB, AC ly tng
ng cc im M, N sao cho AMC =
1
2
ASC, ANB =
1
2
ASB. Gi I l tm ng trn ngoi
tip tam gic AMN. Chng minh rng IS BC.
Li gii
Gi s NB, MC ct li (I) tng ng H, K; HK ct tip tuyn ti A ca (I) V.
S dng nh l Pascal cho su im A, A, H, K, M, N ta thu c V, B, C thng hng.
By gi bi ton kh n gin. rng HIA = 2HNA = ASV suy ra t gic AISV
ni tip.
Do ISV = 180

IAV = 90

.
K n l mt bi ton th v ca Trn Quang Hng, c gii thiu trn din n Math-
Scope.org (Xem [6b])
Bi ton 12. Cho tam gic ABC trc tm H gi H
a
, H
b
, H
c
l im i xng ca H qua
BC, CA, AB; gi d
a
l ng thng Simson ca A tng ng vi H
a
BC, tng t vi d
b
, d
c
th
d
a
, d
b
, d
c
to thnh tam gic thu x vi A

, trong A

, B

, C

l cc chn ng cao k t
Tp ch Ton hc MathVn S 02-2009
A, B, C. (Ghi ch: Hai tam gic c gi l thu x nu ba ng thng ni cc nh tng ng ca
hai tam gic ng quy)
Li gii
S dng nh l Desagues chng ta s ch cn chng minh cc giao im A

, B

, C

ca cc cp
(B

, d
a
), (C

, d
b
), (A

, d
c
) thng hng.
Ch rng theo kt qu bit th H
a
, H
b
, H
c
nm trn ng trn (O) ngoi tip tam gic ABC.
Gi M, S, T tng ng l cc trung im ca BC, OH, AH th th A

, B

, C

, M, T cng thuc
ng trn Euler tm S ca tam gic ABC.
By gi gi D l hnh chiu ca A trn H
a
C. Ta c th thy:
(A

D, A

C) (AD, AC) (AH, AO) (TA

, TM) (mod)
Suy ra DA

l tip tuyn ca (S), d thy DA

chnh l d
a
nn s dng nh l Pascal cho su
im A

, A

, B

, B

, C

, C

ta s thu c A, B, C thng hng, tc l c iu cn chng minh.


III. nh l Pascal vi cc v i cc
nh l Pascal rt th v, cc v i cc cng rt th v v cn mt iu cng rt th v na l
khng t trng hp chng ta cn kt hp hai cng c th v y gii quyt cc bi ton, mt v
d kinh in ca phn ny chnh l php chng minh cho mt nh l rt ni ting ca hnh hc -
nh l Brianchon:
Bi ton 13. Chng minh rng ba ng cho chnh ca mt lc gic ngoi tip ng quy.
Li gii
Tp ch Ton hc MathVn S 02-2009
Ta k hiu ABCDEF l lc gic ngoi tip (O). Tip im ca (O) trn AB, BC, CD, DE, EF, FA
ln lt l M, N, P, Q, R, S.
Xt cc v i cc i vi (O)
Gi I, J, K ln lt l giao im ca cc cp ng thng (SM, PQ), (MN, QR), (NP, RS)
Dng nh l Pascal cho lc gic ni tip MNPQRS ta c I, J, K thng hng, th th cc ng
i cc ca I, J, K ng quy.
D thy cc ng i cc ca I, J, K ln lt l AD, BE, CF nn ta c AD, BE, CF ng quy.
Nh vy ta c iu cn chng minh!
Tip n l hai bi ton kh th v v quan h song song:
Bi ton 14. Cho hnh vung ABCD ngoi tip (O). Tip im ca (O) trn AB, BC, CD, DA
ln lt l M, N, P, Q. Mt im S nm trn cung nh PN ca (O). Tip tuyn ca (O) ti S ct
BC, CD ln lt ti H, K. Chng minh rng MH//AK
Li gii
Tp ch Ton hc MathVn S 02-2009
Xt cc v i cc i vi (O).
Gi s SN ct AB I, SP ct MQ J.
p dng nh l Pascal cho su im M, M, Q, P, S, N ta thu c I, O, J thng hng.
Mt khc ta thy rng I, J ln lt l cc cc ca MH, AK nn MH//AK (v cng vung gc
vi IJ)
Bi ton cui cng ca phn ny l do tc gi xut, v t tng cng tng t bi ton 14
nhng n cn mang mt ngha khc...
Bi ton 15. Cho su im A, B, C, D, E, F cng thuc mt ng trn (O) sao cho ABCD l
hnh ch nht. Gi s EF ct AB, CD ln lt P, Q; BE ct AF H; CE ct DF K. Chng
minh rng PH//QK.
Li gii
Xt cc v i cc i vi (O).
Gi s AE ct BF I, DE ct CF J.
Theo nh l Pascal cho su im A, B, C, F, E, F ta c I, O, J thng hng.
Ta nhn thy I, J chnh l cc cc ca HP, QK nn PH//QK.
Tp ch Ton hc MathVn S 02-2009
By gi gi s cho E trng vo B, ta s thu c ngay mt kt qu rt quen bit l ba ng
cao trong mt tam gic ng quy. Ni cch khc bi ton 15 chnh l mt m rng cho kt qu y.
IV. nh l Pascal v bi ton con bm i vi ng trn
Chng ta chc hn rt quen bit vi kt qu rt p sau ,chnh l bi ton con bm i vi
ng trn.
Bi ton 16. Cho ng trn (O) v dy cung AB. Gi I l trung im ca AB. Qua I v hai
dy cung ty MN v PQ sao cho MP v NQ ct AB ti E, F. Chng minh rng I l trung im
ca EF.
Li gii
Chng ta c kh nhiu chng minh cho kt qu ny, y vi nh l Pascal s dn ra mt cch
tip cn khng ngn nhng rt th v cho n v chng minh ny cn s dng mt b nh sau:
B . Cho bn im phn bit A, B, C, D trong mt phng. Gi s AC ct BD S. Mt ng
thng qua S ct AB, BC, CD, DA ln lt M, N, P, Q. Th th IM = IP khi v ch khi IN = IQ.
B ny thc ra chnh l ni dung ca bi ton con bm i vi cp ng thng v cc bn
c th tm thy mt chng minh cho n ti mc I.39 ca [6c].
Tr li bi ton ban u:
Gi s AP ct MN S; QP ct MB V (Trng hp S, V khng tn ti kh n gin, xin
dnh bn c).
p dng nh l Pascal cho su im A, B, M, N, P, Q ta thu c S, V, F thng hng.
Tip tc s dng b cho bn im S, V, M, P v ng thng AB vi ch IA = IB th s
nhn c kt qu bi ton.
Trn y l mt li gii kh mi m vi bi ton con bm quen bit c tc gi tnh c khm
ph trong khi nh kt hp hai bi ton con bm (tuy nhin kt qu thu c li ch l mt trng
Tp ch Ton hc MathVn S 02-2009
hp ca nh l Pascal). Mnh dn hn, tc gi th tm cch s dng tng trn vo bi ton
con bm m rng ca Klamkin:
Bi ton 17. Cho ng trn (O) vi dy cung AB nhn I lm trung im. Hai im H, K
thuc AB v i xng vi nhau qua I. Gi MN, PQ ln lt l hai dy cung ca (O) i qua H, K.
Gi s QN, MP ct AB ti E, F tng ng. Chng minh rng IE = IF.
Li gii
Vi bi ton ny b trong bi ton 16 t ra khng c hiu lc, chng ta cn n mt b
m rng hn nh sau:
B . Cho bn im phn bit A, B, C, D. Mt ng thng d bt k trong mt phng ct
AB, BC, CD, DA ln lt M, N, P, Q. Th th im I l trung im MP khi v ch khi I l trung
im ca NQ.
B ny c th suy ra trc tip v n gin t nh l Blaikie, bn c c th xem mc I.40
trong [6c].
Tr li vi bi ton 17.
Gi s MN ct AQ V ; QP ct NP T. (Trng hp V, T khng tn ti kh n gin xin
dnh bn c)
S dng nh l Pascal cho su im A, B, M, N, P, Q ta thu c F, T, V thng hng.
Tp ch Ton hc MathVn S 02-2009
n y s dng b cho bn im V, T, Q, N v ng thng AB ta thu c kt qu bi ton.
Trong [7] cng c mt chng minh rt p mt cho bi ton ny bng nh l Pascal, cc bn c
th tm hiu thm.
Bi ton 17 s gip chng ta c mt cch tip cn khc rt th v vi bi ton thch u trn
Tp ch ton tui th II s 25. Xin c trch dn li bi ton bn c tin theo di.
Bi ton 18. Cho ng trn (O); A v B l hai im thuc (O); H l trung im ca AB.
Hai im K, L thuc on AB sao cho HK = HL. im M thuc (O); MH, MK, ML ln lt ct
(O) ti D, E, F. Gi S l giao im ca AB v EF. Chng minh rng SD tip xc vi (O).
Li gii
S dng bi ton s 17 trong trng hp hai im trn ng trn trng nhau vi ch HK = HL
ta nhn c kt qu l...
Nu gi V l giao im ca tip tuyn ti M ca (O) vi AB th HV = HS (V lun tn ti)
Do OV = OS (1)
D thy OM = OD (2)
Ta li c: (V H, V O) (MH, MO) (mod)
Kt hp vi (1) v (2) suy ra rng (OM, OD) (OV, OS) (mod)
V vy (OM, OV ) (OD, OS) (mod) (3)
T (1), (2) v (3) suy ra hai tam gic OMV v ODS bng nhau nn ODS = OMV = 90

Bi ton cui cng l mt bi ton kh p c ng trn Tp ch Ton hc v Tui tr


(Bi T5/297):
Bi ton 19. Cho ng trn tm O ng knh EF. Ly hai im N, P trn ng thng EF
sao cho ON = OP. T im M no nm bn trong ng trn m khng thuc EF, k ng
thng MN ct ng trn ti A v C, ng thng MP ct ng trn ti B v D sao cho B v
O nm khc pha i vi AC. Gi K l giao im ca OB v AC, Q l giao im ca EF v CD.
Chng minh rng cc ng thng KQ, BD v AO ng quy.
Li gii
Tp ch Ton hc MathVn S 02-2009
Gi s AO ct BD I v ct li (O) S; BO ct li (O) T; AB ct EF V .
Theo bi ton 17 s c OV = OQ.
Mt khc ATSB l hnh ch nht nn d thy Q thuc TS.
n y s dng nh l Pascal cho su im A, T, D, S, C, B ta thu c K, I, Q thng hng.
Cc bn hy th m rng bi ton ny nh!
Qua 19 bi ton va ri, tc gi hi vng rng nh l Pascal phn no gn gi hn vi bn
c, cui bi vit l mt s bi tp hay khc lin quan dnh cho cc bn t luyn tp.
Bi tp ngh
Bi 1. Cho tam gic ABC ni tip ng trn (O). Mt ng thng i qua O ct hai cnh
AB, AC ln lt ti M, N. Gi I, J, K ln lot l trung im ca CM, BN, MN. Chng minh bn
im I, J, K, O nm trn mt ng trn.
Bi 2. Cho ng gic li ABCDE c DC = DE v BCD = DEA =

2
. Gi F l mt im nm
trn on thng AB sao cho
AF
BF
=
AE
BC
. Chng minh rng FCE = ADE v FEC = BDC
(Thi v ch quc gia Ba Lan 1997)
Bi 3. (Virgil Nicula) Cho tam gic ABC. ng trn ni tip (I) ca n tip xc vi cc cnh
AB, AC ti E, F tng ng. ng thng EF ct ng trn ng knh BC ti X, Y sao cho
X, F nm v hai pha ca AI.ng trn ct AB, AC tng ng ti M, N. Gi s MX ct NY
ti K; NX ct MY L. Chng minh rng K thuc AI v L thuc HI, trong H l trc tm ca
tam gic ABC.
Bi 4. Mt ng trn ct cc cnh BC, CA, AB ca tam gic ABC ln lt ti D
1
, D
2
; E
1
, E
2
; F
1
, F
2
.
D
1
E
1
ct D
2
F
2
L; E
1
F
1
ct E
2
D
2
M; F
1
D
1
ct F
2
E
2
N. Chng minh rng AL, BM v CN
ng quy. (Chinese Math Olympiad 2005)
Bi 5. Cho tam gic ABC khng cn ni tip ng trn tm O. Gi M, N, P ln lt l trung
im ca BC, CA, AB. Chng minh rng cc ng trn (AOM), (BON), (COP) c hai im chung.
Tp ch Ton hc MathVn S 02-2009
Bi 6. Hy s dng nh l Pascal vi su im khng phn bit chng minh cho nh l
Brianchon.
Bi 7. Cho t gic PQRS ngoi tip ng trn tm O; PR ct QS T. H
A
, H
B
, H
C
, H
D
tng
ng l trc tm ca cc tam gic POQ, QOR, ROS, SOP. Chng minh rng T, H
A
, H
B
, H
C
, H
D
thng hng.
Bi 8. Cho t gic ABCD ngoi tip mt ng trn (O). Tip im ca (O) trn AB, BC, CD, DA
ln lt l M, N, P, Q; BP, BQ ct li (O) tng ng ti F, E. Chng minh rng ME, NF v BD
ng quy. (MOP 1995)
Bi 9. Gi O l tm ca ng trn c cc ng knh BB
t
, CC
t
, M
t
N
t
v cc dy cung
BA
b
, CA
c
. Gi s rng BA
b
, CA
c
ct M
t
N
t
tng ng ti M, N. Gi K
b
, K
c
l giao im th hai
ca NB
t
, NC
t
vi ng trn. Chng minh rng A
b
, A
c
trng nhau khi v ch khi K
b
, K
c
trng nhau.
Bi 10. Xt mt lc gic li ni tip ABCDEF. ng cho BF ct AE, AC ln lt ti M, N.
ng cho BD ct CA, CE ln lt ti P, Q. ng cho DF ct EC, EA ln lt ti R, S. Chng
minh rng MQ, NR v PS ng quy. (Bi T12/344 Tp ch Ton hc v Tui tr).
Ti liu tham kho
[1] Milivoje Lukic, Projective Geometry, Olympiad Traning Materials
Link: http : //www.imomath.com/tekstkut/projg_ml.pdf
[2] Nguyn Minh H, Nguyn Xun Bnh, Bi tp nng cao v mt s chuyn Hnh hc 10,
NXB Gio dc 2006.
[3] V.V.Praxolov, Cc bi ton v hnh hc phng, NXB i hc Quc gia Thnh ph H Ch
Minh.
[4] Tp ch Ton tui th II s 25, 27, 54
[5] Nguyn Phm t, Mt s bi ton s dng nh l Pascal.
[6] Din n MathScope. Link:
http : //forum.mathscope.org/showthread.php?t = 7287
http : //forum.mathscope.org/showthread.php?t = 7198
http : //forum.mathscope.org/showthread.php?t = 4986
[7] Darij Grinberg, On cyclic quadrilaterals and the butterfly theorem
Link: http : //www.cip.ifi.lmu.de/ grinberg/Butterfly.zip
[8] Tp ch Ton hc v Tui tr s 301, 294
[9] Kirans Kedlaya, Geometry Unbound
Tp ch Ton hc MathVn S 02-2009
Link: http : //www math.mit.edu/ kedlaya/geometryunbound/gu 060118.pdf
[10] Mathematical Excalibur vol.10 no. 3, vol. 11 no. 2
Link: http : //www.math.ust.hk/excalibur/
[11] Tuyn tp 30 nm tp ch Ton hc v Tui tr, NXB Gio dc 2004.
[12] Diameters and Chords, Interactive Mathematics Miscellany and Puzzles
Link: http : //www.cut the knot.org/Curriculum/Geometry/DiametersAndChords.shtml
Tp ch Ton hc MathVn S 02-2009
Bt ng thc Turkevici v mt s dng m rng
V Quc B Cn - Sinh vin i hc Y Dc Cn Th, Thnh ph Cn Th
A- M u
Trong bi ny, chng ta s cng bn v bt ng thc ni ting sau:
Bi ton 1. Chng minh rng nu a, b, c, d l cc s thc khng m th
a
4
+ b
4
+ c
4
+ d
4
+ 2abcd a
2
b
2
+ a
2
c
2
+ a
2
d
2
+ b
2
c
2
+ b
2
d
2
+ c
2
d
2
.
Bt ng thc ny c tn l bt ng thc Turkevici do n c nh ton hc Turkevici ngh
u tin trn tp ch Kvant ca Nga vo nm 1979. y l mt bt ng thc p v khng h d
ta gii n. Khng nhng th, n cn n cha nhiu kt qu m rng mnh v th v khin ta
khng th no thot khi s cun ht, ko ta vo vng xoy o su v n. Bi vit ny, chng ti
s gii thiu n cc bn mt s chng minh ca chng ti cho bt ng thc ni ting ny cng
vi mt s m rng ca n.
B - Mt s chng minh
Cch chng minh th nht
Khng mt tnh tng qut, ta gi s 0 a b c d, khi xt hm s sau
f(d) = a
4
+ b
4
+ c
4
+ d
4
+ 2abcd a
2
b
2
a
2
c
2
a
2
d
2
b
2
c
2
b
2
d
2
c
2
d
2
.
Ta c
f

(d) = 4d
3
+ 2abc 2d(a
2
+ b
2
+ c
2
), v f

(d) = 12d
2
2(a
2
+ b
2
+ c
2
) 0,
nn f

(d) l hm ng bin v ta suy ra c


f

(d) f

(c) = 4c
3
+ 2abc 2c(a
2
+ b
2
+ c
2
) = 2c
_
(c
2
b
2
) + a(b a)

0.
Vy f(d) l hm ng bin, suy ra
f(d) f(c) = a
4
+ b
4
+ c
4
+ 2abc
2
a
2
b
2
2(a
2
+ b
2
)c
2
= (c
2
b
2
)
2
+ a(2c
2
ab a
2
)(b a) 0,
nn bt ng thc ca ta c chng minh xong. D thy ng thc xy ra khi v ch khi a = b =
c = d hoc a = 0, b = c = d v cc hon v tng ng.
Nhn xt. Cch gii ny da trn ng dng ca phng php kho st hm s vo chng minh
bt ng thc. y l mt phng php hay v c ng dng kh nhiu gii ton bt ng
thc. Mt iu lu khi ta la chn hm s kho st l hy ch n ng thc ca bi
ton, chng hn bi ton ny, vi gi thit a b c d th ngoi b (a, b, c, d) (1, 1, 1, 1) ra,
ng thc cn xy ra ti a = 0, b = c = d nn ta nn chn hm f(d) nh trn tin cho vic kho st!
Cch chng minh th hai
Vi gi thit rng 0 a b c d, ta thy rng kt qu ca bi ton l hin nhin da trn
ng thc sau
a
4
+ b
4
+ c
4
+ d
4
+ 2abcd a
2
b
2
a
2
c
2
a
2
d
2
b
2
c
2
b
2
d
2
c
2
d
2
=
= (c d)
2
(a + c + d)(c + d a) + (d
2
b
2
)(c
2
b
2
) + a(b a)(2cd ab a
2
).
Tp ch Ton hc MathVn S 02-2009
Cch chng minh th ba
y l mt li gii bng phng php dn bin ca Gabriel Dospinescu [1]. t x = a
2
, y = b
2
,
z = c
2
, t = d
2
, khi bt ng thc ca ta c vit li l
f(x, y, z, t) = x
2
+ y
2
+ z
2
+ t
2
+ 2

xyzt xy xz xt yz yt zt 0.
Do tnh i xng nn ta c th gi s mt cch khng mt tng qut rng t = min{x, y, z, t} . Vi
gi thit ny, ta s chng minh f(x, y, z, t) f(u, u, u, t) 0, vi u =
3

xyz. Tht vy, ta thy rng


bt ng thc ny tng ng vi
x
2
+ y
2
+ z
2
xy yz zx t (x + y + z 3u) ,
hay l
x
2
+ y
2
+ z
2
xy yz zx t (x + y + z 3
3

xyz) .
Do t
3

xyz nn ta ch cn chng minh c


x
2
+ y
2
+ z
2
xy yz zx
3

xyz (x + y + z 3
3

xyz) ,
tng ng
x
2
+ y
2
+ z
2
+ 3
3
_
x
2
y
2
z
2

xyz(x + y + z) + xy + yz + zx.
p dng bt ng thc Schur dng bc 3 cho b (x
2/3
, y
2/3
, z
2/3
), ta c
x
2
+ y
2
+ z
2
+ 3
3
_
x
2
y
2
z
2

cyc
x
2/3
y
2/3
(x
2/3
+ y
2/3
).
Mt khc, theo bt ng thc AM GM th ta c

cyc
x
2/3
y
2/3
(x
2/3
+ y
2/3
) 2(xy + yz + zx),
v

cyc
x
2/3
y
2/3
(x
2/3
+ y
2/3
) =

cyc
x
4/3
(y
2/3
+ z
2/3
)
2x
4/3
y
1/3
z
1/3
+ 2y
4/3
z
1/3
x
1/3
+ 2z
4/3
x
1/3
y
1/3
= 2
3

xyz(x + y + z).
Cng tng ng 2 bt ng thc ny ri chia c 2 v cho 2, ta thu c

cyc
x
2/3
y
2/3
(x
2/3
+ y
2/3
)
3

xyz(x + y + z) + xy + yz + zx.
T kt hp vi bt ng thc Schur trn, ta c th d dng thu c
x
2
+ y
2
+ z
2
+ 3
3
_
x
2
y
2
z
2

xyz(x + y + z) + xy + yz + zx,
hay ni cch khc, bt ng thc f(x, y, z, t) f(u, u, u, t) c chng minh. V vi bt ng thc
ny, ta thy rng chng minh bt ng thc cho, ta ch cn chng minh c f(u, u, u, t) 0,
tc l t
2
+2u

ut 3ut, y l mt kt qu hin nhin ng theo AM GM. Php chng minh ca


ta c hon tt.
Nhn xt. y l mt cch dn bin rt c sc, thay v nh cc php dn bin thng thng
a v 2 bin bng nhau trc ri tip tc lp lun th tc gi li gii dn mt cch trc tip
a v ba bin bng nhau gii. Cch dn bin ny kh mnh v s dng n, ta c gii c mt
s kt qu tng t nh sau
Tp ch Ton hc MathVn S 02-2009
1. Nu a, b, c, d l cc s khng m c tng bng 4 th
3(a
2
+ b
2
+ c
2
+ d
2
) + 4abcd 16.
(Nguyn Anh Cng)
2. Nu a, b, c, d l cc s khng m c tng bng 1 th
abc + bcd + cda + dab
1
27
+
176
27
abcd.
(Nguyn Minh c, IMO Shortlist 1996)
Cch chng minh th t
Tng t vi li gii 3, ta cng s chng minh bt ng thc sau vi x, y, z, t l cc s khng m
x
2
+ y
2
+ z
2
+ t
2
+ 2

xyzt xy + yz + zx + xt + yt + zt,
tng ng
x
2
+ y
2
+ z
2
+ t
2
+ 2

xyzt (x + z)(y + t) + xz + yt,


hay l
1
2
(x z)
2
+
1
2
(y t)
2
+
1
2
(x + z y t)
2

_
xz

yt
_
2
.
By gi, ta gi s x z y t, v t x = t + m, y = t + n, z = t + p, vi m p n 0, th bt
ng thc trn c th c vit li thnh
1
2
(mp)
2
+
1
2
n
2
+
1
2
(m + p n)
2

_
_
(t + m)(t + p)
_
t(t + n)
_
2
.
Nu n + p m th ta c 0
_
(t + m)(t + p)
_
t(t + n)

mp. Tht vy, bt ng thc bn
tri l hin nhin nn ta s chng minh bt ng thc bn phi, ta vit li n nh sau
_
(t + m)(t + p)
_
t(t + n) +

mp.
Bnh phng 2 v v thu gn, ta thy bt ng thc ny tng ng vi tng bt ng thc trong
dy sau
(t + m)(t + p) t(t + n) + mp + 2
_
tmp(t + n),
t(mn + p) 2
_
tmp(t + n),
t(mn + p)
2
4mp(t + n),
t(m
2
+ n
2
+ p
2
2mn 2mp 2np) 4mnp.
Bt ng thc ny hin nhin ng bi v
m
2
+ n
2
+ p
2
2mn 2mp 2np = m(mn p) + n(n m) + p(p m) 2np 0.
Do , khng nh ca ta trn l ng, v s dng n, ta c th a bt ng thc v chng minh
1
2
(mp)
2
+
1
2
n
2
+
1
2
(m + p n)
2
mp, hay m
2
+ n
2
+ p
2
mn + np + pm.
Bt ng thc ny hin nhin ng theo AM GM.
Nu m n + p th ta c 0
_
(t + m)(t + p)
_
t(t + n)
m+pn

2
. Tht vy, bt ng thc
ny tng ng vi
_
(t + m)(t + p)
m + p n

2
+
_
t(t + n),
Tp ch Ton hc MathVn S 02-2009
hay l
(t + m)(t + p)
(m + p n)
2
2
+ t(t + n) + (m + p n)
_
2t(t + n),
t(m + p n) + mp
(m + p n)
2
2
+ (m + p n)
_
2t(t + n).
Ta c (m + p n)
_
2t(t + n) (m + p n)t

2 (m + p n)t, v
(m + p n)
2
2
mp =
1
2
m(m2n) +
1
2
(n p)
2
0 (do m n + p 2n),
nn bt ng thc trn l ng, t ta suy ra c
_
_
(t + m)(t + p)
_
t(t + n)
_
2

1
2
(m + p n)
2

1
2
(mp)
2
+
1
2
n
2
+
1
2
(m + p n)
2
.
Bt ng thc ca ta c chng minh xong.
Cch chng minh th nm
Khng mt tnh tng qut, ta gi s a b c d, khi vi ch rng

cyc
a
4
4abcd = (a
2
b
2
)
2
+ (c
2
d
2
)
2
+ 2(ab cd)
2
,
v
3

cyc
a
4
2

sym
a
2
b
2
=

sym
(a
2
b
2
)
2
,
ta c th vit li bt ng thc cn chng minh nh sau
(a
2
c
2
)
2
+ (b
2
c
2
)
2
+ (a
2
d
2
)
2
+ (b
2
d
2
)
2
2(ab cd)
2
.
By gi, p dng cc bt ng thc Cauchy Schwarz v AM GM, ta c
(a
2
d
2
)
2
+ (b
2
d
2
)
2

1
2
(a
2
+ b
2
2d
2
)
2

1
2
(2ab 2d
2
)
2
= 2(ab d
2
)
2
2(ab cd)
2
,
nn bt ng thc trn l hin nhin, v php chng minh ca ta c hon tt.
Cch chng minh th su
Ta s gi s a b c d v vit li bt ng thc cn chng minh nh sau
a
4
+ b
4
+ c
4
a
2
b
2
b
2
c
2
c
2
a
2
d
_
d(a
2
+ b
2
+ c
2
d
2
) 2abc

,
hay
(a
2
b
2
)
2
+ (b
2
c
2
)
2
+ (c
2
a
2
)
2
2d
_
d(a
2
+ b
2
+ c
2
d
2
) 2abc

.
Do (a
2
b
2
)
2
2d(a + b)(a b)
2
, (b
2
c
2
)
2
2d(b + c)(b c)
2
, (c
2
a
2
)
2
2d(c + a)(c a)
2
nn
(a
2
b
2
)
2
+ (b
2
c
2
)
2
+ (c
2
a
2
)
2
2d
_
(a + b)(a b)
2
+ (b + c)(b c)
2
+ (c + a)(c a)
2

.
T ta c th a bi ton v chng minh
(a + b)(a b)
2
+ (b + c)(b c)
2
+ (c + a)(c a)
2
d(a
2
+ b
2
+ c
2
d
2
) 2abc,
Tp ch Ton hc MathVn S 02-2009
tng ng
2(a
3
+ b
3
+ c
3
+ abc) ab(a + b) bc(b + c) ca(c + a) d(a
2
+ b
2
+ c
2
d
2
).
By gi, p dng bt ng thc Schur bc 3, ta c ab(a+b)+bc(b+c)+ca(c+a) a
3
+b
3
+c
3
+3abc,
nn chng minh bt ng thc ny, ta ch cn chng minh
2(a
3
+ b
3
+ c
3
+ abc) (a
3
+ b
3
+ c
3
+ 3abc) d(a
2
+ b
2
+ c
2
d
2
),
tng ng
a
3
+ b
3
+ c
3
abc d(a
2
+ b
2
+ c
2
d
2
).
p dng bt ng thc Chebyshev, ta c
a
3
+ b
3
+ c
3

1
3
(a + b + c)(a
2
+ b
2
+ c
2
).
Do , ta cn chng minh
1
3
(a + b + c)(a
2
+ b
2
+ c
2
) abc d(a
2
+ b
2
+ c
2
) d
3
,
hay l
(a
2
+ b
2
+ c
2
)(a + b + c 3d) 3(abc d
3
).
p dng bt ng thc AM GM, ta c
3(abc d
3
) = 3
_
3

abc d
__
3

a
2
b
2
c
2
+ d
3

abc + d
2
_
9
3

a
2
b
2
c
2
_
3

abc d
_
3(a
2
+ b
2
+ c
2
)
_
3

abc d
_
3(a
2
+ b
2
+ c
2
)
_
a + b + c
3
d
_
= (a
2
+ b
2
+ c
2
)(a + b + c 3d),
nn bt ng thc trn l hin nhin ng. Php chng minh ca ta c hon tt.
Cch chng minh th by
Khng mt tnh tng qut, gi s rng a b c d 0. Khi , t
A = a
4
+ b
4
+ c
4
a
2
b
2
b
2
c
2
c
2
a
2
=

a,b,c
(a
2
b
2
)(a
2
c
2
),
B = d
2
(ab + bc + ca) d
2
(a
2
+ b
2
+ c
2
) = d
2

a,b,c
(a b)(a c),
v
C = d
4
+ 2abcd d
2
(ab + bc + ca),
ta thu c
a
4
+ b
4
+ c
4
+ d
4
+ 2abcd a
2
b
2
a
2
c
2
a
2
d
2
b
2
c
2
b
2
d
2
c
2
d
2
=
= A + B + C =

a,b,c
(a
2
b
2
)(a
2
c
2
) d
2

a,b,c
(a b)(a c) + C
=

a,b,c
(a
2
d
2
)(a b)(a c) + (ab + bc + ca)

a,b,c
(a b)(a c) + C.
V a
2
d
2
b
2
d
2
c
2
d
2
0 nn

a,b,c
(a
2
d
2
)(a b)(a c) (a
2
d
2
)(a b)(a c) + (b
2
d
2
)(b c)(b a)
(b
2
d
2
)(a b)(a c) + (b
2
d
2
)(b c)(b a)
= (b
2
d
2
)(a b)
2
0.
Tp ch Ton hc MathVn S 02-2009
Ngoi ra, ta d dng kim tra c

a,b,c
(a b)(a c) 0.
Do vy, chng minh bt ng thc Turkevici, ta thy rng ch cn chng minh C 0 l , tc
l
d
3
+ 2abc d(ab + bc + ca) 0.
Ta c
d
3
+ 2abc d(ab + bc + ca) = d
3
dab + c(2ab ad bd)
d
3
dab + d(2ab ad bd)
= d(d a)(d b) 0.
Nn bt ng thc C 0 l hin nhin, v v th, php chng minh ca ta c hon tt.
Nhn xt. Theo chng ti, hai li gii 5 v 6 l hai li gii p v c sc nht cho bi ton
ny, bi l chng ch hon ton s dng nhng bt ng thc kinh in nh AM GM, Cauchy
Schwarz v Schur. Chng cho ta thy rng d hin nay c nhiu phng php mnh gii bt
ng thc n u i na th cc li gii s dng bt ng thc kinh in vn l nhng li gii p
v su sc nht. y chnh l v p ca s "th s" m hiu qu. Ngoi 7 cch chng minh ny,
cn c mt s cch chng minh khc nh cch chng minh dng phng php dn bin mnh SMV
ca Phm Kim Hng, dng phng php phn tch bnh phng ca Michael Rozenberg hay dng
phng php EV ca Vasile Cirtoaje, . . . Nhng chng ti cho rng nhng li gii ny u phi
dng n nhng cng c qu mnh, v chng khin bi ton mt i v p ca n. V th chng ti
s khng gii thiu chng y, nu cc bn c hng th mun tham kho thm nhng li gii ny
th c th lin h trc tip vi chng ti. By gi chng ta s n vi nhng m rng ca bi ton ny
C- Mt s m rng
rng bt ng thc Turkevici tng ng vi
3

cyc
a
4
+ 4abcd
_

cyc
a
2
_
2
. (1)
Vi nhng dng pht biu nh th ny, ta thng ngh n liu bt ng thc ca ta c th tng
qut cho n bin c khng? May mn thay, iu l c trong trng hp ny. V ta c kt
qu sau
Bi ton 2. Cho x
1
, x
2
, . . . , x
n
(n 2) l cc s thc khng m. Khi ta c
(n 1)
n

i=1
x
2
i
+ n
n
_
x
2
1
x
2
2
x
2
n

_
n

i=1
x
i
_
2
.
Chng minh ca kt qu ny, cc bn c th tham kho thm [1].
By gi, nu ta p dng bt ng thc Cauchy Schwarz
_

cyc
a
2
_
2

cyc
a
__

cyc
a
3
_
,
Tp ch Ton hc MathVn S 02-2009
a bt ng thc v chng minh
3

cyc
a
4
+ 4abcd
_

cyc
a
__

cyc
a
3
_
.
Rt may mn l kt qu ny li mt ln na ng, v n chnh l mt trng hp ring ca bi
ton sau (c Suranyi ngh trong cuc thi Miklos Schweitzer Competition)
Bi ton 3. Chng minh rng nu x
1
, x
2
, . . . , x
n
l cc s khng m th
(n 1)(x
n
1
+ x
n
2
+ + x
n
n
) + nx
1
x
2
x
n
(x
1
+ x
2
+ + x
n
)(x
n1
1
+ x
n1
2
+ + x
n1
n
).
(Suranyi)
Cc bn cng c th tham kho thm chng minh ca Gabriel Dospinescu [1].
By gi, chng ta s i n kt qu m rng chnh m chng ti mun gii thiu n cc bn
phn ny
Bi ton 4. Cho x
1
, x
2
, . . . , x
n
(n 2) l cc s thc khng m. Chng minh rng vi mi s thc
k, ta lun c
(n 1)
n

i=1
x
n+k
i
+ x
1
x
2
x
n
n

i=1
x
k
i

_
n

i=1
x
i
__
n

i=1
x
n+k1
i
_
.
Ta thy rng kt qu m Suranyi ngh thc cht chnh l mt trng hp ring ca kt qu
ny (ng vi k = 0). chng minh n, ta s thc hin php quy np trn n tng t nh Gabriel
Dospinescu thc hin [1] chng minh bt ng thc Suranyi.
Vi n = 2 th bt ng thc trn tr thnh ng thc, cn vi n = 3 th sau mt vi bin i,
ta thy rng n tng ng vi bt ng thc Schur dng bc 3
x
k+1
1
(x
1
x
2
)(x
1
x
3
) + x
k+1
2
(x
2
x
3
)(x
2
x
1
) + x
k+1
3
(x
3
x
1
)(x
3
x
2
) 0,
nn n hin nhin ng. By gi, gi s rng bt ng thc trn ng vi n bin, ta s chng minh
rng n cng ng vi n + 1 bin. Tht vy, do tnh thun nht nn khng mt tnh tng qut, ta
c th gi s x
1
+ x
2
+ + x
n
= n, khi ta cn chng minh
n
_
n

i=1
x
n+k+1
i
+ x
n+k+1
n+1
_
+ x
n+1
_
n

i=1
x
i
__
n

i=1
x
k
i
+ x
k
n+1
_
(n + x
n+1
)
_
n

i=1
x
n+k
i
+ x
n+k
n+1
_
.
p dng gi thit quy np, ta c
(n 1)
n

i=1
x
n+k
i
+
_
n

i=1
x
i
__
n

i=1
x
k
i
_
n
n

i=1
x
n+k1
i
,
nn ta ch cn chng minh c
n
_
n

i=1
x
n+k+1
i
+ x
n+k+1
n+1
_
+ nx
n+1
n

i=1
x
n+k1
i
(n 1)x
n+1
n

i=1
x
n+k
i
+
_
n

i=1
x
i
_
x
k+1
n+1
(n + x
n+1
)
_
n

i=1
x
n+k
i
+ x
n+k
n+1
_
,
Tp ch Ton hc MathVn S 02-2009
tng ng
n
_
n

i=1
x
n+k+1
i

n

i=1
x
n+k
i
_
nx
n+1
_
n

i=1
x
n+k
i

n

i=1
x
n+k1
i
_
+
+ x
k+1
n+1
_
n

i=1
x
i
nx
n1
n+1
+ (n 1)x
n
n+1
_
0.
+ Nu n + k 1 0 th theo bt ng thc Chebyshev, ta c
n

i=1
x
n+k
i

n

i=1
x
n+k1
i
=
n

i=1
x
n+k
i

1
n
_
n

i=1
x
i
__
n

i=1
x
n+k1
i
_
0.
Gi s x
1
x
2
x
n
x
n+1
(ta c th gi s iu ny v bt ng thc cn chng minh
l i xng), suy ra 0 x
n+1
1. Khi
n
_
n

i=1
x
n+k+1
i

n

i=1
x
n+k
i
_
nx
n+1
_
n

i=1
x
n+k
i

n

i=1
x
n+k1
i
_

n
_
n

i=1
x
n+k+1
i

n

i=1
x
n+k
i
_
n
_
n

i=1
x
n+k
i

n

i=1
x
n+k1
i
_
= n
n

i=1
(x
n+k+1
i
2x
n+k
i
+ x
n+k1
i
) = n
n

i=1
x
n+k1
i
(x
i
1)
2
0.
Mt khc, p dng bt ng thc Bernoulli (ch rng
xixn+1
xn+1
0 i = 1, 2, . . . , n), ta c
n

i=1
x
i
= x
n
n+1
n

i=1
_
1 +
x
i
x
n+1
x
n+1
_
x
n
n+1
_
1 +
n

i=1
x
i
x
n+1
x
n+1
_
= nx
n1
n+1
(n 1)x
n
n+1
,
nn hin nhin
n

i=1
x
i
nx
n1
n+1
+ (n 1)x
n
n+1
0.
Do , bt ng thc cho cng ng vi n + 1 bin. Theo nguyn l quy np, ta suy ra
c n ng vi mi n 2.
+ Nu n + k 1 0 th theo bt ng thc Chebyshev, ta c
n

i=1
x
n+k
i

n

i=1
x
n+k1
i
=
n

i=1
x
n+k
i

1
n
_
n

i=1
x
i
__
n

i=1
x
n+k1
i
_
0.
Gi s x
1
x
2
x
n
x
n+1
(ta c th gi s iu ny v bt ng thc cn chng minh
l i xng), suy ra x
n+1
1. Khi
n
_
n

i=1
x
n+k+1
i

n

i=1
x
n+k
i
_
nx
n+1
_
n

i=1
x
n+k
i

n

i=1
x
n+k1
i
_

n
_
n

i=1
x
n+k+1
i

n

i=1
x
n+k
i
_
n
_
n

i=1
x
n+k
i

n

i=1
x
n+k1
i
_
= n
n

i=1
(x
n+k+1
i
2x
n+k
i
+ x
n+k1
i
) = n
n

i=1
x
n+k1
i
(x
i
1)
2
0.
Tp ch Ton hc MathVn S 02-2009
V theo bt ng thc Bernoulli (vi ch rng 0
xixn+1
xn+1
1), ta c
n

i=1
x
i
= x
n
n+1
n

i=1
_
1 +
x
i
x
n+1
x
n+1
_
x
n
n+1
_
1 +
n

i=1
x
i
x
n+1
x
n+1
_
= nx
n1
n+1
(n 1)x
n
n+1
,
nn hin nhin
n

i=1
x
i
nx
n1
n+1
+ (n 1)x
n
n+1
0.
Do , bt ng thc cho cng ng vi n + 1 bin. Theo nguyn l quy np, ta suy ra
c n ng vi mi n 2.
Php chng minh ca ta c hon tt.
Vi li gii ca kt qu m rng trn, chng ti xin c kt thc bi vit ca mnh y. Rt
mong nhn c nhng kin ng gp qu bu ca bn c cho bi vit ny. Xin chn thnh cm
n!
Ti liu tham kho
[1] T. Andreescu, V. Cartoaje, G. Dospinescu, M. Lascu, Old and New Inequalities, Vol. 1, GIL,
2004.
[2] V. Q. B. Can, C. Pohoat , Old and New Inequalities, Vol. 2, GIL, 2008.
[3] I. Boreico, V. Q. B. Can, Mircea Lascu, Yong Su, Bin Xiong, Introduction to Inequalities, GIL,
2009.
[4] T. N. Dung, G. Dospinescu, Mixing variables method, chuyn bi dng gio vin THPT
chuyn, h 2005.
Tp ch Ton hc MathVn S 02-2009
Cc phng php tnh tch phn
Nguyn Vn Vinh, Sinh vin chuyn ngnh Ton L, i hc Tng hp Quc gia Belarus
Ni n vn v tch phn th ngi hc ton u va chm t nhiu, tuy nhin c nhng ngi
ch mc s cp vi mc tnh ton thng thng c nhiu ngi li s dng n nh l cng c
gii quyt cc vn phc tp hn, nhng gc ng dng no cng cn c nhng k nng v
s hiu bit nht nh. Bi vit nh ny nu ra mt s phng php m nhiu ngi lm ton cao
cp hay s dng, c th ni ch l nhng chia s mang tnh c nhn trong qu trnh hc tp thu
nhn c. Cc phng php tnh tch phn thng thng xin khng nu li, ch cp cc phng
php nh s dng php bin i Laplace, Fourier, hm sinh ca cc hm c bit, tch phn tham
s, hm Gamma, Beta, Gauss, Dirac, l thuyt thng d, tch phn kp, chui Taylor... Lng kin
thc c s dng trong bi ny kh nhiu khng th trnh by tt c, nn ch c th im qua i
nt kin thc cn thit trc mi phng php, c th c nhiu khi nim nh ngha c b qua.
V khi lng bi vit kh di nn cc v d a ra ch mang tnh nh hnh phng php c cc
bi tp p dng km theo.
1. Php bin i Laplace, v bin i Fourier
Nh chng ta bit L[f(t), p] =

_
0
f(t)e
pt
dt l nh ca hm f(t) qua php bin i Laplace
vi i s p. Trong f(t) c gi l hm gc v cho bi cng thc
f(t) = L
1
[f(t), p] =
1
2i
c+i
_
ci
L[f(t), p] e
pt
dp
i vi php bin i Fourier th c nhiu dng khc nhau nhng c hai dng m chng ta hay
gp v s dng l dng i xng
F [f(t), p] =
1

f(t)e
ipt
dt
V khng i xng

F [f(t), p] =

f(t)e
ipt
dt
Tng ng vi hai hm gc c cho bi cng thc
f(t) = F
1
[f(t), p] =
1

2
+
_

F [f(t), p] e
ipt
dp
V
f(t) =

F
1
[f(t), p] =
1
2
+
_

F [f(t), p] e
ipt
dp
Cc tnh cht ca hai php bin i ny kh nhiu mi ngi c th xem thm cun The Trans-
forms and Applications Handbook ca Alexander D. Poularikas.
Tp ch Ton hc MathVn S 02-2009
n gin ta k hiu cc hm nh khng c i s L[f(t)] , F [f(t)] ,

F [f(t)]
Chng ta bt u bng mt v d n gin.
V d 1. Tnh I(x) =

_
0
1 cos xt
t
2
dt
Li gii
Ni chung, cch p dng ca c hai php bin i ny l thay v ta tnh trc tip cc tch phn
cn tnh th ta c th tc ng vo c hai v mt php bin i Laplace hay Fourier v sau ta
tm hm gc ca tch phn va tm c th s thu c kt qu nh mong mun.
Ta c
L[I(x)] =

_
0
e
px
_
_

_
0
1 cos xt
t
2
dt
_
_
dx =

_
0
1
t
2
_
_

_
0
e
px
(1 cos xt) dx
_
_
dt
=

_
0
1
t
2
L[1 cos xt] dt =

_
0
1
t
2
_
1
p

p
p
2
+ t
2
_
dt =
1
p
arctg
t
p

t=0
=

2p
2
Ta d dng nhn thy L
1
_

2p
2
_
=

2
x.
Vy ta c I(x) =

2
x.
Ta p dng tng t cho php bin i Fourier v thu c kt qu tng t.
Bi tp p dng
1. Tnh I(x) =

_
0
cos xt
a
2
+ t
2
dt
2. Tnh I =

_
0
xsinmx
1 + x
2
dx
3. Tnh I =

_
0
cos t. sintx
t
dt
Bn cnh ta c th s dng cc tnh cht ca php bin i Laplace, Fourier thu c cc
kt qu nhanh v hiu qu. C ba tnh cht n gin m chng ta bit khi lm quen vi php
bin i Laplace
i.

_
0
f(t)dt
t
=

_
0
L[f(t)] dp
ii. L
_
f(t)
t
_
=

_
p
L[f(t), q] dq
iii. L
_
_
t
_
0
f(x)g(t x)dx
_
_
= L[f] .L[g]
Tp ch Ton hc MathVn S 02-2009
Ta th xem thm mt vi v d ng dng ba tch cht ny
V d 2. Tnh I =

_
0
sint
t
dt
Li gii
Tch phn ny kh l phc tp khi ta s dng cc kin thc thng thng ngay c s dng tch
phn tham s cng kh di nhng nu p dng tnh cht trn ta thy ngay kt qu
I =

_
0
sint
t
dt =

_
0
L[sint] dp =

_
0
dp
1 + p
2
= arctgp|

0
=

2
V d 3. Tnh I =

_
x
p
2
1
(p
2
+ 1)
2
dp
Li gii
Da vo tnh cht th hai d dng thu c kt qu ca bi trn:
I = L[cos t, x] =
x
1 + x
2
V d 4. Tnh I =
t
_
0
e
x
sinxcos(t x)dx
Li gii
p dng tnh cht th ba trn ta c
L
_
_
t
_
0
e
x
sinxcos(t x)dx
_
_
= L[e
x
sinx] L[cos x] =
1
(p 1)
2
+ 1
.
p
p
2
+ 1
=
1
5
_
p
p
2
+ 1

p 1
(p 1)
2
+ 1
2
1
p
2
+ 1
+ 3
1
(p 1)
2
+ 1
_
T ta c ly nghch nh ca hm thu c v thu c kt qu cn tm kh d dng v cc
hm nh c dng quen thuc
I =
1
5
_
cos t e
t
cos t 2 sint + 3e
t
sint
_
Bi tp p dng
1. Tnh I =
t
_
0
e
t
ch()d, trong l hng s.
2. Tnh I =

_
0
e
t
sint
t
dttrong > 0.
3. Tnh I =

_
x
(p a)
_
(p a)
2
+
2
_
2
dp
Tp ch Ton hc MathVn S 02-2009
Php bin i Laplace cn c cc tnh cht khc m mi tnh cht u c th p dng x l linh
hot cc tch phn nh tnh cht v dch nh, cng thc Duhamel,... (bn c c th tham kho
thm [2],[5]). Vi php bin i Fourier, ngoi k thut nu trn i vi php bin i Laplace
ta cn c nhng cch p dng khc.
V d 5. Tnh
I() =
+
_

e
x
2
dx, > 0
Li gii
cho thun tin khi cc bn c cc sch tham kho thm v d dng ta s s dng php bin
i Fourier khng i xng c dng

F [f(t)] =
+
_

f(t)e
2ipt
dt
Nhn thy

F
_
e
x
2
_
=

F
_
e
(

x/

)
2
_
=
_

2
p
2
/
T d dng thu c tch phn cn tnh
I() =
+
_

e
x
2
e
2i0x
dx =

F [0] =
_

e
0
=
_

y chnh l tch phn Euler Poisson m chng ta bit.


Vic p dng cng thc trn tnh tch phn ht sc n gin v thun tin tuy nhin trong
mt s trng hp c nhiu hm th vic la chn hm gc cho php bin i Fourier cng ht sc
quan trng, hu ht cc hm lin quan n tch phn ny l tch phn c cha hm lng gic, hm
m, hm lu tha,...
V d 6. Tnh
I() =
+
_

sin(x) cos (2x)


x
dx, < <
Li gii
Ta c

F [sinc(x)] =
_
_
_
1, |p| < 1/2
1/2, |p| = 1/2
0, |p| > 1/2
Trong sinc
_
x

_
=
sinx
x
.
Khi ta c tch phn cn tnh l
I() =
+
_

sin(x)
x
e
2ix
dx =

F [] ==
_
_
_
1, || < 1/2
1/2, || = 1/2
0, || > 1/2
Tp ch Ton hc MathVn S 02-2009
i vi php bin i Fourier bn cnh cc tnh cht c bn gn ging vi php bin i Laplace
th cn c mt s cc tnh cht c bit khc. Mt vi tnh cht thng hay s dng:
+ ng thc Plancherel
+
_

|f(x)|
2
dx =
+
_

F [p]

2
dp
+ ng thc Parseval
+
_

f(x)g(x)dx =
+
_

F [p]

G[p]dp
V d 7. Tnh
I =
+
_

dx
(1 + x
2
)
2
Li gii
Ta c

F
_
1
1 + x
2
_
= e
2|p|
p dng ng thc Plancherel, th th
I =
+
_

1
1 + x
2

2
dx = 2
2
+
_

e
4p
dp =

2
Ta thu c kt qu rt nhanh v nh nhng.
V d 8. Tnh
I =
+
_

_
sinx
x
_
3
dx
Li gii
p dng ng thc Parseval i vi hai hm
f(x) =
sinx
x
, g(x) =
_
sinx
x
_
2
Ta thu c tch phn cn tm
I =
+
_

_
sinx
x
_
3
dx =
+
_

F
_
sinc
_
x

__
G
_
sinc
2
_
x

__
dp =
2
1
2
_
1 +
1
2
_
=
3
4

Trong sinc
_
x

_
=
sinx
x
Php bin i Fourier cng c tnh cht ca tch chp ging vi php bin i Laplace (tnh cht
s 3).
Tp ch Ton hc MathVn S 02-2009
S dng tnh cht ny ta cng d dng tnh c tch phn sau
g(x) =
+
_

sinu
u
sin (x u)
(x u)
du = sinc(x)
Bi tp p dng
1. Tnh I =
+
_

cos (x)
1 + x
4
dx
2. Tnh I =
+
_

_
sinx
x
_
3
dx
3. Tnh I =
+
_

_
sinx
x
_
4
dx
4. Tnh I =
+
_
0
sin(ax) cos (bx)
x
dx
5. Tnh I =
+
_

sin(ax)
x(x
2
+ b
2
)
dx
Php bin i Laplace v Fourier cn cha kh nhiu cc tnh cht hu ch cho vic tnh tch
phn. Ngay c i vi cc tch phn bi khi s dng phng php ny cng ht sc hiu qu. Php
bin i Laplace v Fourier cng c cc dng suy rng cho nhiu bin khc nhau m vi mi dng
iu c nhng ng dng ht sc th v. V bi vit ch mang tnh gii thiu cc phng php nn
hi vng s tr li ch ny trong mt bi vit khc.
2. Khai trin tch phn thnh chui
y l mt k thut kh s cp nhng rt th v v thng gp trong cc bi ton tnh tch phn
phc tp. Vn la chn hm khai trin s quyt nh bi gii c p v ti u hay khng. Khi
khai trin v hon v tch phn ca tng v tng ca tch phn ta cn ch n cc i tng thu
c c quen v m bo tnh hi t ca tch phn hay khng.
V d 9. Tnh I =

_
0
e
x
_
_
x
_
0
e
t
1
t
dt
_
_
lnxdx
Li gii
y l mt bi ton kh kh ca tp ch Crux. Tuy nhin nu p dng k thut khai trin chui
Taylor th ta cng thu c kt qu khng my l kh khn. Di y l li gii khi p dng k
thut trn
I =

_
0
e
x
_
_
x
_
0
e
t
1
t
dt
_
_
lnxdx =

_
0
e
x
_
_
_
_
x
_
0

n=0
(t)
n
n!
1
t
dt
_
_
_
_
lnxdx
Tp ch Ton hc MathVn S 02-2009
=

n=1
(1)
n
n!n

_
0
e
x
_
_
x
_
0
t
n1
dt
_
_
lnxdx =

n=1
(1)
n
n!n

_
0
e
x
x
n
lnxdx
=

n=1
(1)
n

(n + 1)
n!n
=

n=1
(1)
n
(n + 1)
n
Trong (x), (x) l cc hm Gamma v PolyGamma.
Nh ta thy trn, ta s dng k thut khai trin hm e
t
di dng chui Taylor v chuyn
tch phn cn tnh thnh tng ca chui.
Vn tnh tng ca chui chng ta khng bn y nhng ta cng d dng nhn c kt qu
ca chui ny l

n=1
(1)
n
(n + 1)
n
= ln2 +
1
_
0
ln2 + ln(1 + t)
1 t
dt =
1
12
_

2
+ 12 ln2 + 6 ln
2
2
_
Trong l hng s Euler-Mascheroni.
Chng ta c mt bi bin i tch phn thnh chui kh th v tp ch La Gaceta.
V d 10. Chng minh
I =
1

1
_
1
xe
2x

1 x
2
dx =

n=0
1
n! (n + 1)!
Li gii
Ta c
I =
1

1
_
0
x
_
e
2x
e
2x
_
dx

1 x
2
=
1

1
_
0

n=0
(2x)
2n+2
(2n + 1)!
dx

1 x
2
=
1

n=0
2
2n+2
(2n + 1)!
1
_
0
x
2n+2
dx

1 x
2
=
1

n=0
2
2n+2
(2n + 1)!
/2
_
0
sin
2n+2
d
=
1

n=0
2
2n+2
(2n + 1)!
(2n + 2)!
2
2n+3
((n + 1)!)
2
=

n=0
1
n! (n + 1)!
Nhn vo dng biu din ca chui, ta nhn thy ngay l hm I
1
(2) (Hm modified Bessel
bc nht).
Phng php trn cho php ta tip cn nhiu tch phn phn kh v gip ch kh nhiu cho cng
tc nghin cu. Mt trong nhng ng dng mnh v ht sc hiu qu l nh l cng, nhng
ngi lm ton cao cp v lin quan n cc vn v phng trnh vi tch phn hn s thy y
l mt cng c hiu qu v th v. Cc bn c th xem thm v vn ny [7].
S dng k thut ny ta tnh c kh nhiu tch phn hay m nhiu phng php khc khng
lm c hoc qu cng knh, th d nh mt bi ca tp ch PIMU
1
_
0
ln(1 + x)
x
dx =

n=1
(1)
n1
1
_
0
x
n1
n
dx =

n=1
(1)
n1
n
2
=

2
12
Tp ch Ton hc MathVn S 02-2009
Bn cnh mt php chuyn tch phn v chui rt thng gp khi gii cc bi ton tch phn
lin quan n phn l v phn nguyn. Xin a ra mt v d v vn ny chng ta thy r s
th v ca phng php chuyn tch phn thnh chui.
Chng ta bt u bng mt v d nh vi hm phn nguyn
V d 11. Tnh
1
_
0
__
2
x
_
2
_
1
x
__
dx
Li gii
y l bi ton c bn p dng k thut ni trn. S dng k thut khai tch phn thnh
dng chui ta c:
1
_
0
__
2
x
_
2
_
1
x
__
dx =

k=1
1/k
_
1/(k+1)
__
2
x
_
2
_
1
x
__
dx
=

k=1
_
1
k + (1/2)

1
k + 1
_
Vn tnh tng chui trn khng kh khn, kt qu cui cng ta thu c

k=1
_
1
k + (1/2)

1
k + 1
_
= 2
_
1
3

1
4
+
1
5

1
6
+ ...
_
= 2 ln2 1 0.386
Bi ton kh n gin nhng phn no ni ln c k thut ta m t v c ngha kh th
v, gi tr tch phn ta va tnh c chnh l xc sut s d khng b hn mt na ca s b
chia, ni r hn ta xt phng trnh ng d n n

(mod) , 0 n

< , th gi tr tch phn thu


c l xc sut xy ra trng hp n



2
.
Ta xem tip mt v d hay v phn l
V d 12. Tnh I =
1
_
0
_
1
x
_
lnxdx
Li gii
y l mt bi ton rt hay tp ch MJMS v di y l li gii ca Ovidiu Furdui.
Ta d dng bin i tch phn v dng sau
I =
1
_
0
_
1
x
_
lnxdx =

k=1
k+1
_
k
lnt
t
2
(t k) dt
Tch phn tng phn cc tch phn thu c, ta c
I =

k=1
_
ln
2
(k + 1) ln
2
k
2
+ ln(k + 1) lnk
ln(k + 1)
k + 1

1
k + 1
_
Ch rng = lim
n
n

k=1
1
k
lnn l hng s Euler-Mascheroni, v
1
= lim
n
_
n

k=1
ln k
k

ln
2
n
2
_
l hng s Stieltjes.
Tp ch Ton hc MathVn S 02-2009
Ta thu c tng ca chui trn l
I = +
1
1.
Cc cng on bin i thu c kt qu kh n gin bn c c th lm nh l bi tp.
S dng k thut nh trn ta thu c nhiu kt qu p ca cc tch phn c gi tr c bit
l cc tham s nh Stieltjes, Euler-Mascheroni, Gold ratio...
Cng dng phng php trn, Ng Phc Nguyn Ngc tnh c tch phn sau rt p v
thu c mt biu din kh th v ca hng s Euler-Mascheroni
I =
1
_
0
_
1
x
_
x
1 x
dx = .
Chng ta nhn thy trn vic chuyn cc tch phn cn tnh n cc chui tng ng
gip ta thu c nhiu ng thc p. Bng phng php ny chng ta cng c th chng minh
hai dng biu din ca hm hypergeometric function l tng ng nhau
F(a, b, c, z) =
(c)
(b)(c b)
1
_
0
t
b1
(1 t)
cb1
(1 tz)
a
dt =

k=0
(a)
k
(b)
k
(c)
k
z
k
k!
Chng minh xin dnh cho bn c.
Bi tp p dng
1. I =
1
_
0
{lnx} x
m
dx, m > 1.
2. I =
1
_
0
lnxln
2
(1 x)
x
dx.
3. I =
1
_
0
_
1
x
_
x
1 x
dx = .
4. I =
1
_
0
1
_
0
x
t
y
t
_
x
y
_
_
y
x
_
dxdy, t > 1.
3. Tch phn ph thuc tham s
C th ni y l mt ch rt ln vi tnh ng dng rt cao. C kh nhiu k thut i vi
php tnh tch phn nhng phng php tch phn tham s l mt vn ht sc th v. u kh
khn nht khi p dng phng php ny bn phi ngh ra tham s cn thm l g, chng ta i theo
chiu thun hay ngc ca php bin i. Vic p dng phng php ny cho php ta thu c
nhiu kt qu rt l th v lin quan n chui, hm c bit. V ch ny kh di v c nhiu
ng dng phc tp nn s c cp chi tit trong bi vit s tp ch tip theo.
Tp ch Ton hc MathVn S 02-2009
Ti liu tham kho
1. Manzirov A.V., Polyanin A.D.; Tuyn tp phng trnh tch phn: Cc phng php gii ; NXB
Factorial, Moskva 2000 (bn Ting Nga).
2. Alexander D. Poularikas, The Transforms and Applications Handbook, Second Edition; CRC
Press, 2000.
3. L Vn Trc, Nguyn Vn Tho, Phng php Ton cho Vt l ; NXB HQG H Ni.
4. Cc tp ch CMJ, Crux, MJMS, PIMU, SSMJ.
5. Tuyn tp MathVn - Cc k thut bin i vi tch phn, 2009 (preprint)
6. A.M. Mathai, Hans J. Haubold; Special Functions for Applied Scientists; Springer, 2008.
7. Erofeenko V.T., Tuyn tp v nh l cng; NXB Khoa hc v K thut, Minsk 1989 (bn
Ting Nga).
Tp ch Ton hc MathVn S 02-2009
L thuyt cc qun xe
Nguyn Tun Minh, Lp C nhn Cht lng cao, Kha III - i hc Hu
A - M u
Hon v vi cc v tr cm
Ta c mt m hnh cho mt bi ton tng qut nh sau:
Xt cc tp khc rng X
1
, X
2
, ..., X
n
{1, 2, .., n} v k hiu S
n
tp cc hon v vi di bng n
t P(X
1
, X
2
, ..., X
n
) = { S
n
| (i) / X
i
, i = 1, 2, ..., n }, tp X
i
c gi l v tr cm
ca (i), cc hon v thuc P(X
1
, X
2
, ..., X
n
) gi l hon v vi cc v tr cm (permutation with
forbidden positions) tng ng vi h (X
1
, X
2
, ..., X
n
)
Mt hon v S
n
tng ng vi mt cch sp t n con xe trn bn c vua n n cc ta
(i, (i)) ( y ta nh s cc ct v cc dng bng cc s 1, 2, ..., n t tri sng sang phi v t
trn xung di, (x, y) l ta ca nm ct th x v hng th y), d nhin l (i) = (j) nn
khng c 2 con no n nhau.
Nu P(X
1
, X
2
, ..., X
n
) th hon v ny tng ng vi mt cch sp t n con xe ln bn c
nn sao cho khng c hai con no n nhau v con xe nm ct th i th khng c php t vo
cc vung c ta thuc tp M
i
= {(i, x) | x X
i
}, cc v tr ny gi l v tr cm (d nhin
M
i
M
j
= vi i = j).
Gi A
i
l tp cc cch sp xp m con xe ct th i c t vo v tr cm. Theo nguyn l
bao hm-loi tr ta c kt qu tng qut
|P(X
1
, X
2
, ..., X
n
)| = |S
n
| |A
1
A
2
... A
n
|
= n!
n

i=1
|A
i
| +

1i<jn
|A
i
A
j
| .... + (1)
n
|A
1
A
2
... A
n
|
Gi r
k
l s sp t k con xe ln bn c n n sao cho mi con xe u v tr cm, quy c
r
0
= 1. Th th

1i1<i2<...<i
k
n
|A
i1
A
i2
... A
i
k
| = r
k
(n k)!
Do
|P(X
1
, X
2
, ..., X
n
)| =
n

k=0
(1)
k
r
k
(n k)!
S cch sp t cc qun xe
Gi r
k
(C) l s cch sp t k con xe (rook number) ln min vung C (min y khng
nht thit l phi lin thng) trn bn c sao cho khng c 2 con no n nhau.
Hm sinh R
C
(x) =

k=0
r
k
(C)x
k
gi l a thc xe (rook polynomial) ca min C, quy c
r
0
(C) = 1.
Tp ch Ton hc MathVn S 02-2009
Mt min cc vung C trn bn c m n c th c c trng bi mt trn (c
ij
)
mn
trn
trng F
2
= {0, 1} vi c
ij
bng 1 nu (i, j) C v c
ij
= 0 nu (i, j) / C.
Mt s tnh cht quan trng:
1. R
C
(x) = R
C\
(x) + xR
C
(x) trong l mt vung trong C, C \ l min vung
nhn c t C khi b i v C l min vung nhn c t C khi b i tt c cc c
cng hng v cng ct vi .
Tht vy, khi sp xp k qun xe ln min C, trong c nh, th c hai trng hp xy ra:
- c sp mt qun xe, khi i vi k 1 qun xe cn li th khng th sp cng hng
hoc cng ct vi . S cch sp trong trng hp ny l R
k1
(C ).
- khng c sp qun xe no, khi i vi k qun xe c th sp trn min C. S cch
sp xp trong trng hp ny l R
k
(C \ ). V vy:
R
k
(C) = R
k
(C \ ) +R
k1
(C )
T y d suy ra kt qu vi dng hm sinh.
Min cc vung S gi l mt block ca min cc vung C nu tha mn:
i. Vi bt k cc dng i, i

c cha ca S v ct j khng cha no ca S th c


ij
= c
i

j
.
ii. Vi bt k dng i khng cha no ca S, v cc ct j, j

cha ca S th c
ij
= c
ij
.
Ta c mt m rng (xem thm [1]) ca tnh cht 1 nh sau:
2. Vi C l min vung trn bn c v S l mt block nm trn s hng v t ct th th
R
C
(x) =
min(s,t)

j=0
r
j
(S)x
j
R
C(S,j)
(x)
Trong C(S, j) vi 0 j min(s, t) l min nhn c bng cch b cc ca C sao cho:
i. B i tt c cc ca S
ii. B i tt c cc thuc j dng trong s s dng cha cc ca S
iii. B i tt c cc thuc j ct trong s t ct cha cc ca S
Chng minh ny cng khng kh khn, chi tit xin dnh cho bn c.
3. R
C1C2
(x) = R
C1
(x)R
C2
(x) vi C
1
v C
2
l hai min khng c hng no chung v ct no
chung, C
1
C
2
l min vung bao gm tt c cc vung ca C
1
v C
2
Tht vy, v C
1
, C
2
l hai min khng c hng no chung v ct no chung nn mi cch sp t
i qun xe ln C
1
v j qun xe ln C
2
s ng vi mi cch sp t i + j qun xe ln min C
1
C
2
,
vi i, j 0. V vy r
k
(C
1
C
2
) =

i+j=k
r
i
(C
1
)r
j
(C
2
). T y nhn c kt qu di dng hm sinh.
Gi s B l mt min vung trn bn c v B l min b ca n trn bn c mn, th th s
cch t k qun xe ln B s bng s cch sp t k qun xe ln bn c v coi cc thuc B nh l
cc v tr cm. Tng t nh phn u, k hiu A
i
vi i = 1, 2, ..., k l s cp sp t m qun xe
Tp ch Ton hc MathVn S 02-2009
th i nm vo min cm l B.
Bng quy tc nhn ta tnh c

1i1<i2<...<isk
|A
i1
A
i2
... A
is
| = r
s
(B).
(ms)!(n s)!
(n k)!(n k)!(k s)!
y ta ch rng s cch sp t qun xe ln bn c pq bng
p!q!
(p t)!(q t)!t!
, vi t min(m, n)
(chng minh iu ny xem nh bi tp), v ta p dng cho t = k s, p = ms, q = n s.
Theo nguyn l bao hm-loi tr, ta c
r
k
(B) =
k

s=0
(1)
s
(ms)!(n s)!
(n k)!(n k)!(k s)!
r
s
(B)
T y ta c mt lin h p gi a thc xe ca hai min vung b nhau:
4.
R
B
(x) =
n

k=0
_
k

s=0
(1)
s
(ms)!(n s)!
(n k)!(n k)!(k s)!
r
s
(B)
_
x
k
Bi tp.
1. K hiu R
m,n
(x) l a thc xe ca bng ch nht mn . Chng t rng
R
m,n
= R
m1,n
(x) +xnR
m1,n1
(x)
v dng tng t
R
m,n
= R
m,n1
(x) +xmR
m1,n1
(x)
T suy ra
R
m,n
(x) =
min(n,m)

k=0
m!n!
(mk)!(n k)!k!
x
k
2. K hiu L

n
=
e
x
n!x

d
n
dx
_
x
n+
e
x
_
(a thc Laguerre). Chng t rng
R
n,n+
(x) = n!x
n
L

n
_

1
x
_
3. Chng minh rng

m=0
R
m,n
(x)
t
m
m!
= (1 +xt)
n
e
t
v

m=0

n=0
R
m,n
(x)
t
m
u
n
m!n!
= e
t+u+xtu
4. Vi C l mt bng vung bt k, ta vit r
k
thay v r
k
(C). Gi s n l bc ca a thc R
C
(x),
chng cc bt ng thc sau
a.
_
k +m
k
_
r
k+m
r
m
r
k
Tp ch Ton hc MathVn S 02-2009
b. k
k2
r
k

_
r
2
k 1
_
vi 1 k n.
c. r
k1
r
k+1
r
2
k
d. r
k1
r
k+1
_
1 +
1
k
__
1 +
1
n k
_
r
2
k
e.
_
r
k
_
n
k
_
_
1/k

_
r
m
_
n
m
_
_
1/m
vi 0 m < k n.
f. r
k
n
nk
vi 0 k
n
2
g.
n k
k + 1
r
k
r
nk1
vi 0 k
n1
2
L thuyt cc qun xe (Rook Theory) m i tng ca n l a thc xe c nghin cu
u tin bi Kaplansky and Riordan vo nm 1946, v sau l cc m rng ca Goldman vi s
ng dng ca nhiu phng php t hp hin i t nhng nm 1970. Trong nhng nm gn y
Haglund t c nhiu thnh cng trong vic gn kt a thc xe vi nhiu lnh vc khc nh chui
siu hnh hc, bi ton m ma trn trn trng hu hn, l thuyt biu din nhm. L thuyt cc
qun xe c quan h gn gi vi nhiu ng dng trong l thuyt th, ngi ta cng vn dng
a thc xe cng vi c hc lng t v i s Weyl. Cn trong T hp m ni ring, a thc xe
lin quan n hng lot cc bi ton m v hon v, phn hoch, hnh vung Latin...
Trong phn m u chng ta lm quen vi cch tnh ton ma thc xe vi s quy ca cc
min vung. Phn ng dng my tnh trong tnh ton a thc xe s c cp cui bi vit,
xem nh l ph lc.
B - M rng v ng dng
1. Bi ton m hon v bt ha
Bi ton 1. (Derangement problem
1
) Tm s cc hon v S
n
sao cho (i) = i.
Li gii
Bi ton tng ng vi vic tm s cc cch sp xp cc qun xe ln bn c n n vi v
min cc v tr cm chnh l ng cho chnh ca bn c:
C = {(1, 1), (2, 2), ..., (n, n)}
R rng min ny c th c xem l phn hoch thnh n vung i mt khng cng hng v
cng ct. a thc xe ca mi nh vy l R
0
(x) = 1 +x.
p dng tnh cht 3, ta c R
C
(x) = [R
0
(x)]
n
= (1 +x)
n
. Suy ra r
k
(C) =
_
n
k
_
.
p dng bi ton hon v vi cc v tr cm, ta thu c cng thc tnh s cc hon v S
n
tha mn iu kin bi ton:
D
n
=
n

k=0
(1)
k
_
n
k
_
(n k)! = n!
n

k=0
(1)
k
k!
1
Bi ton c nghin cu u tin bi Pierre Raymond de Montmort vo nm 1713
Tp ch Ton hc MathVn S 02-2009
Mt hon v nh vy gi l v trt t (derangement permutation).
Ch l khi n th
D
n
n!
e
1
.
Bi ton 2. (Mnage problem
2
) Tm s cch sp xp n cp c du, ch r vo mt bn trn 2n
ch (n 3) sao cho cc c du, ch r ngi lun phin nhau, nhng khng xy ra trng hp ch
r ngi bn cnh c du ca mnh.
Li gii
u tin, ta sp n c du vo bn, sao cho gia hai c du trng mt gh dnh cho mt ch
r no . S cch sp xp nh vy bng 2.n!
Trong mi cch sp xp cc c du, nh s h theo chiu kim ng h ln lt l 1, 2, ..., n. Gh
trng bn phi c du th i ta nh s l i.
By gi ta xp cc ch r vo n gh trng ny sao cho tha mn yu cu bi ton.
Gi s ch r ca c du th i c sp vo gh s (i) {1, 2, ..., n}.
Th th (i) = i, i + 1 (mod n)
Ta c th cho tng ng mt hon v S
n
(n 2) nh vy bng vi mt cch sp cc qun
xe ln bn c n n vi min cc v tr cm l:
C
n
= {(1, 1), (2, 2), ..., (n, n), (1, 2), (2, 3), ..., (n, 1)}
2
Bi ton c nghin cu u tin bi douard Lucas vo nm 1891
Tp ch Ton hc MathVn S 02-2009
p dng tnh cht 1, i vi min vung C
n
khi b i (n, 1) ta c min S
n
, nu xa i
dng v ct cha ny ta c min S
n1
. Ta c:
R
Cn
(x) = R
Sn
(x) +xR
Sn1
(x)
Vi min S
n
ta loi i (1, 1) c min T
n
, loi i hng v ct cha ny thu c S
n1
,
tng t ta nhn c
R
Sn
(x) = R
Tn
(x) +xR
Sn1
(x)
Vi min T
n
, ta loi i (1, 2) thu c min S
n1
, loi i hng v ct cha ny thu c
T
n1
. Th th
R
Tn
(x) = R
Sn1
(x) +xR
Tn1
(x)
T cc quan h quy trn, ta rt ra c:
R
Cn+1
(x) (2x + 1)R
Cn
(x) +x
2
R
Cn1
(x), n = 2, 3, ...
V vi cc trng hp u tin, ta tnh c R
C2
(x) = 1+4x+2x
2
, R
C3
(x) = 1+6x+9x
2
+2x
3
.
Kt qu cui cng, tnh c (c th s dng hm sinh hoc a thc c trng):
R
Cn
(x) =
n

k=0
2n
2n k
_
2n k
k
_
x
k
p dng bi ton hon v vi cc v tr cm, ta c s cc hon v S
n
tha mn iu kin l
U
n
=
n

k=0
(1)
k
2n
2n k
_
2n k
k
_
(n k)!
y l s cch sp xp n ch r vo n v tr trng nh nh s. Cng thc ca s cch sp
xp cc c du ch r tha mn iu kin bi ton l:
2.n!
n

k=0
(1)
k
2n
2n k
_
2n k
k
_
(n k)!
Trng hp tng qut ca Bi ton 1 v Bi ton 2, ta gi hon v S
n
sao cho
(i) = i, i + 1, ..., i + k 1 (mod n) l k-bt ha (k-discordant). Cho n nay ngi ta c
cc kt qu cho bi ton ny cho k = 1, 2, 3, 4, 5. Trng hp k = 3, 4, 5 c th tip cn mt cch s
cp tm quan h quy v hm sinh, trong bi vit khng cp n do kh phc tp, bn c
c th tham kho [2], [3], [4]. Trong trng hp tng qut, vn tm mt cng thc chnh xc
tnh s hon v k-bt ha vn l bi ton m.
Bi tp.
1. Mt hon v S
n
c gi l khng lin tip (nonconsecutive) nu nh tha mn (i) = i+1
vi i = 1, 2, ..., n 1. Tm cng thc Q
n
tnh s cc hon v di n khng lin tip v chng t
Q
n
= D
n
+D
n1
2. Chng minh rng:
a. D
n
= (n 1)(D
n1
+D
n2
), n 1
b. D
n
= nD
n1
+ (1)
n
, n 2
Tp ch Ton hc MathVn S 02-2009
c. D
n
=
_
n!
e
+
1
2
_
, n 1
d. D
n
=
_
n!
e
+
n + 2
(n + 1)
2
_
, n 2
e. D
n
=
_
(e +e
1
)n!

[en!] , n 2
f. D
n
=
__
[e(n +m2)!]
(n +m2)!
+
n +m
(n +m1)(n +m1)!
+e
1
_
n!
_
[en!]
vi m 3, n 2
g. D
n
=
__
{e(n + 2m)!}
(n + 2m)!
+
m

i=1
n + 2i 1
(n + 2i)!
+e
1
_
n!
_
vi m 1, n 2.
3. Tm cng thc tnh s cc cch sp xp 2n qun xe ln bn c 2n 2n sao cho khng c hai
con no nm trn hai ng cho ln ca bn c.
4. Vi U
n
=
n

k=0
(1)
k
2n
2n k
_
2n k
k
_
(n k)! (mnage number), chng minh rng:
a. U
n
= nU
n1
+
n
n 2
U
n2
+
4(1)
n1
n 2
, n 2
b. U
n
= nU
n1
+ 2U
n2
(n 4)U
n3
U
n4
, n 4
2. Bng Ferrers
Vi 1 c
1
c
2
... c
n
th min cc vung c xc nh bi
F = {(i, j) | 1 i n, 1 j c
i
}
gi l bng mt bng Ferrers. Nh vy bng Ferrers c to thnh t dy cc ct vung vi
chiu cao khng gim.
Ta c cc nh ngha:
1. h(F) = (c
1
, c
2
, ..., c
n
) gi l vector chiu cao ca F.
2. h
m
(B) = (c
(m)
1
, c
(m)
2
, ..., c
(m)
m
) (s n) vi c
(m)
i
= 0, i = 1, 2, ..., m n v c
(m)
i
= c
im+n
vi
i = mn + 1, ..., n gi l m-vector chiu cao ca F.
Ch rng h
n
(F) = h(F). C th hiu mt cch n gin rng h
m
(F) l vector to ra t h(F)
bng cch thm cc gi tr 0 vo mn v tr u tin .
3. s
m
(F) = (s
(m)
1
, s
(m)
2
, ..., s
(m)
m
) vi s
(m)
i
= c
(m)
i
i + 1 gi l vector m-cu trc ca F.
D dng kim chng cc tnh cht sau:
i. s
(m)
1
0
ii. s
(m)
i
s
(m)
i1
1 vi i = 2, 3, ..., m
Tp ch Ton hc MathVn S 02-2009
iii. s
i
(m) = s
(m)
i1
1 khi v ch khi c
(m)
i
= c
(m)
i1
Ngc li, nu mt vector gm n s nguyn tha mn (i) v (ii) th n l mt vector n-cu trc
ca mt bng Ferrers xc nh duy nht.
4. P
m
(F, x) =

m
k=0
r
k
(F).[x]
mk
(m s) gi l a thc m-giai tha ca bng Ferrers F.
nh l 1. (Factorization theorem) Xt bng Ferrers F vi n ct, vi m n c vector m-cu
trc s
m
= (s
(m)
1
, s
(m)
2
, ..., s
(m)
m
) v a thc m-giai tha P
m
(F, x) th th
P
m
(F, x) =
m

i=1
(x +s
(m)
i
)
y [x]
i
= x(x 1)(x 2)...(x i + 1) nu 0 i x v bng 0 nu i > x.
Chng minh
Gi s F c vector chiu cao l h(F) = (c
1
, c
2
, ..., c
n
) v m-vector chiu cao
h
m
(F) = (c
(m)
1
, c
(m)
2
, ..., c
(m)
m
), m n
Vi x N, xt bng F
x,m
vi vector chiu cao
h(F
x
) = (c

1
, c

2
, ..., c

m
) = (x, ..., x, x +c
1
, x +c
2
, ..., x +c
n
)
D thy, v phi ca ng thc cn chng minh ng bng r
m
(F
x,m
).
Ta s tnh r
m
(F
x,m
) theo mt cch khc. Coi F
x,m
= F C, C l bng hnh ch nht x m
nm bn di F (hnh v)
Trong mi trng hp sp xp m qun xe ln F
x,m
, gi s trn F c k qun xe, th s cch sp
xp cc qun xe trn F bng r
k
(F), khi cn li mk qun xe sp xp trn C. Sau loi i k ct
b chim bi k qun xe sp trn F th ch cn mk ct c th sp xe ln c trn C. Nn
i vi mk qun xe cn li ta xem nh sp xp chng ln mt min ch nht x (mk), c tt
Tp ch Ton hc MathVn S 02-2009
c [x]
mk
cch sp xp.
V vy r
m
(F
x,m
) = P
m
(F, x) =
m

k=0
r
k
(F).[x]
mk
ng thc cn chng minh c kim chng vi v hn im l cc s t nhin, hn na do
dng a thc nn ta thu c kt qu cho mi x R.
H qu 1. Xt bng Ferrers F vi h(F) = (s + 1, 2(s + 1), ..., (n 1)(s + 1)) th th
P
n
(F, x) = x(x +s)(x + 2s)...(x + (n 1)s)
H qu ny suy ra t nh l 1 vi h
n
(F) = (0, s + 1, 2(s + 1), ..., (n 1)(s + 1)), v tng ng
s
n
(F) = (0, s, 2s, ..., (n 1)s).
Trng hp c bit, i vi bng F = (1, 2, ..., n 1) th th r
k
(F) = S(n, n k), y S(s, r)
l s Stirling loi II.
Tht vy, p dng h qu trn ta c
n

k=1
r
k
[x]
nk
= x
n
y l phng trnh quen thuc ca s Stirling loi hai.
Ngoi ra chng ta c th gii quyt bng mt phng n thun ty hn. S(n, nk) l s cc phn
hoch tp {1, 2, ..., n} thnh nk khi khc rng khng giao nhau. Xt mi cch sp xp k qun xe
ln bng F = {(i, j) | 1 i n, 1 j i}. Qun xe c sp v tr (i, j) th ta cho tng ng hai
s i, j nm cng mt khi trong phn hoch. Vic kim tra y l mt song nh xin dnh cho bn c.
Ta nh ngha rng hai min vung l tng ng (rook equivalence) nu chng c cng a
thc xe. Theo nh l 1 ta suy ra c rng:
i. Hai bng Ferrers F
1
v F
2
tng ng nu v ch nu vi m no th a thc m-giai tha
ca chng ng nht nhau.
ii. Hai bng Ferrers F
1
v F
2
tng ng nu v ch nu vi m no th S
m
(F
1
) = S
m
(F
2
).
y k hiu S
m
(F) l tp c lp gm cc thnh phn ca vector s
m
(F).
nh l 2. Mi bng Ferrers tng ng vi duy nht mt bng Ferrers vi dy chiu cao cc
ct l tng ngt.
Chng minh
Trc ht, xt bng Ferrer F
0
vi cc ct c chiu cao tng ngt 1 a
1
< a
2
< ... < a
n
.
Th th h
m
(F
0
) = (0, 0, ..., 0, a
1
, ..., a
n
), trong m n v tr u tin bng 0. Khi s
m
(F
0
) =
(0, 1, 2, ..., (mn 1), u
1
, u
2
, ..., u
n
) vi (mn 1) u
1
... u
n
. Mt vector m-cu trc
nh vy xc nh duy nht mt bng Ferrers, v bng ny c cc ct c chiu cao tng ngt.
Vi F l mt bng Ferrers bt k, theo nhn xt trn ta s xy dng mt bng Ferrers F

tng
ng vi F, ngha l c m S
m
(F) = S
m
(F

), v cc thnh phn s
m
(F

) c sp tng t nh
dng ca s
m
(F
0
).
Ta c r
1
(F) = |F| nn cc bng tng ng th c cng s . Xt m = |F| +1, th th s
(m)
1
= 0.
Gi s t l thnh phn c gi tr nh nht trong s
m
(F). Theo tnh cht ca ca vector m-cu trc
Tp ch Ton hc MathVn S 02-2009
th 1, 2, ..., (t 1) cng xut hin trong s
m
(F). Xt vector:
q = (0, 1, 2, ..., (t 1), t, q
1
, q
2
, ..., q
mt1
)
y q
i
l cc phn t ca tp c lp S
m
(F) loi i cc phn t 0, 1, 2, ..., t v sp th t
khng gim q
1
q
2
... q
mt1
. Nh vy q l mt vector m-cu trc ca mt bng Ferrers F

vi
cc ct c chiu cao tng ngt. Hn na do t l phn t nh nht trong tp c lp S
m
(F) = S
m
(F

)
nn F

l bng Ferrers vi cc ct c chiu cao tng ngt duy nht tng ng vi F.


Mt bi ton ny sinh l c bao nhiu bng Ferrers tng ng vi mt bng Ferrers cho trc?
Ta thy rng s
m
(F) v s
m
(F

) u l cc hon v ca tp c lp S
m
(F). Nh vy bi ton c
gii quyt khi ta m c s hon v ca tp c lp S
m
(F).
nh l 3. S bng Ferrers tng ng vi bng Ferrers F cho trc bng:
_
j
0
+j
1
1
j
1
__
j
1
+j
2
1
j
2
_
...
_
j
t1
+j
t
1
j
t
_
y t l thnh phn c gi tr b nht trong vector s
|F|+1
(F), v j
i
l s ln xut hin ca
phn t i trong tp c lp S
|F|+1
(F).
Chng minh cng thc ny c th xem trong [8], [9].
Bi tp.
1. Cho (c
n
) l dy tng bao gm cc s nguyn dng, v bng Ferrers F
n
vi h
F
= (c
1
, c
2
, ..., c
n
).
t
q
n
(x) = x
cn+1
e
x
R
Fn
_
1
x
_
Chng t rng
q
n
(x) = x
cn+1cn
d
dx
q
n1
(x)
2. C bao nhiu bng Ferrers tng ng vi bng Ferrers F, trong :
a. h(F) = (1, 2, ..., n 1).
b. h(F) = (2, 4, ..., 2(n 1)).
c. h(F) = (k, 2k, ..., (n 1)k) vi k > 2.
C - Lp trnh tnh ton
1. S dng Mathematica
Bi ton ca chng ta l tm a thc xe trn mt min vung cho trc.
Lp hm RookPolynomial nh sau sinh a thc xe
RookPolynomial[n_List] :=
Module[{stacklist, top, SameColumnValues, SameRowValues,
EntriesToEliminate, InclusionList, ExclusionList},
stacklist = {Union[n]};
While[Union[Flatten[stacklist]] =!= {1, x}, top = First[stacklist];
Tp ch Ton hc MathVn S 02-2009
SameColumnValues = Position[top, {_, top[[1, 2]]}];
SameRowValues = Position[top, {top[[1, 1]], _}];
EntriesToEliminate = Union[SameColumnValues, SameRowValues];
InclusionList = Append[Delete[top, EntriesToEliminate], {x}];
ExclusionList = Delete[top, 1];
If[Length[ExclusionList] == 0, ExclusionList = {{1}}];
stacklist = Delete[Append[Append[stacklist, ExclusionList], InclusionList], 1];
While[Union[Flatten[First[stacklist]]] == {x},
stacklist = RotateLeft[stacklist]]];
Total[Apply[Times, stacklist, 2]]]
Input ca hm ny l mt danh sch, bao gm ta cc ca min vung cn tnh ton a
thc xe.
Th d. Tnh R
C
(x) vi min vung C = {(1, 1), (2, 2), (3, 3), (4, 4), (5, 5)}.
Ta s dng hm trn nh sau:
RookPolynomial[{{1, 1}, {2, 2}, {3, 3}, {4, 4}, {5, 5}}]
Nhp Shift+Enter, kt qu s l
Out[1]= 1 + 5 x + 10 x^2 + 10 x^3 + 5 x^4 + x^5
2. S dng Maple
Nh phn u, ta cng c th lp mt gii thut tng t bng Maple. Nhng trnh s lp
li, ta xt bi ton tm a thc xe trn bn c n n vi min cc v tr cm cho trc.
u tin ta s dng hm complement_board tnh min cho php t cc qun xe trn bn
c, gi tr tr v l mt danh sch.
> with(combinat);
> complement_board := proc (F, n)
local i, j, x, L;
L := [];
for i to n do
for j to n do
if not(member([i, j], F)) then L := [op(L), [i, j]] end if
end do
end do;
RETURN(L)
end proc;
Hm non_attacking_rooks_pos tm tt c cc cch sp t k qun xe trn bn c n n, vi
min cc v tr cm cho di dng mt danh sch F.
>non_attacking_rooks_pos := proc (F, k, n)
local i, j, ans, cols, R, pos, F0, F1, F2, m, P, x, stop0, is;
P := permute(n, n);
F0 := F; pos := [];
if k = 1 then RETURN(complement_board(F0, n)) end if;
for x in P do
stop0 := 0;
R := choose(n, k);
Tp ch Ton hc MathVn S 02-2009
for is in R do
stop0 := 0;
for i in is do
if member([i, x[i]], F0) then stop0 := 1 end if
end do;
if stop0 = 0 then pos := [op(pos), [seq([i, x[i]], i = is)]] end if
end do
end do;
RETURN(convert(convert(pos, set), list))
end proc
Cui cng, hm rook_polynomial tnh a thc xe theo bin x ca bi ton sp xp cc qun
xe trn bn c n n vi min cc v tr cm l danh sch F.
>rook_polynomial := proc (F, n, x)
local i, a, p;
p := 1+add(nops(non_attacking_rooks_pos(F, i, n))*x^i, i = 1 .. n);
RETURN(p)
end proc
Th d. Sp xp cc qun xe ln bn c 5 5 vi min cc v tr cm l
F = {(1, 1), (2, 2), (3, 3), (4, 4), (5, 5)}
Ta lm nh sau:
> rook_polynomial([[1, 1], [2, 2], [3, 3], [4, 4], [5, 5]], 5, x);
Nhp Enter kt qu s l
1 + 20 x + 130 x^2 + 320 x^3 + 265 x^4 + 44 x ^5
Ch l y hm non_attacking_rooks_pos cho gi tr tr v l mt danh sch cc cch
sp xp.
Ta c th xut ra tt c cc cch sp xp trong tng trng hp ca n, k v F.
> array_rooks := proc (L, n)
local i, j, A;
A := array(1 .. n, 1 .. n);
for i to n do
for j to n do
A[i, j] := 0;
if member([i, j], L) then A[i, j] := R end if
end do
end do;
print(A)
end proc
Th d.
L:=non_attacking_rooks_pos([[1,2],[1,3],[2,1],[2,3],
[3,2],[3,4],[3,5],[5,4]],5,5):nops(L);
for i from 1 to 4 do array_rooks(L[i],5); end do;
Tp ch Ton hc MathVn S 02-2009
Khi kt qu trn mn hnh s l s cc cch sp xp 5 qun xe ln bn c 5 5 vi min cc
v tr cm nh trn l bng 18, v 4 cch sp xp u tin:
D nhin lit kt ht ta thay 4 trong vng for bng nops(L).
Gii thut cng p dng c i vi cc s sp xp cc qun xe ln cc bng vung khng c
v tr cm.
Th d. a thc xe ca cc bng 4 4, 5 5
Ta lm nh sau:
rook_polynomial([],4,x);
rook_polynomial([],5,x);
Kt qu l:
1 + 16 x + 72 x^2 + 96 x^3 + 24 x^4
1 + 25 x + 200 x^2 + 600 x^3 + 600 x^4 + 120 x^5
Tp ch Ton hc MathVn S 02-2009
Ti liu tham kho
[1] Abigail Mitchell, A block decomposition algorithm for computing rook polynomials; preprint
at http://arxiv.org/abs/math/0407004v1
[2] William Oscar Jules Moser, The number of very reduced 4 n Latin rectangles; Canad. J.
Math. 19 (1967) pp. 1011-1017.
[3] Earl Glen Whitehead Jr, Four-discordant permutations; Journal of the Australian Mathemat-
ical Society (Series A)(1979), 28 : 369-377.
[4] Joseph M. Santmyer, Five discordant permutations; Graphs and Combinatorics (1993) 9:279-
292.
[5] K.P. Kohas, Rook numbers and Rook Polynomials; MCCME, Moskva - 2003 (in Russian).
[6] Richard P. Stanley, Enumerative Combinatorics, Volume 1; Cambridge University Press, 1999.
[7] Mehdi Hassani, Derangements and Applications; Journal of Integer Sequences, Vol. 6 (2003).
[8] Jay R. Goldman, J. T. Joichi, Dennis E. White, Rook Theory. I: Rook Equivalence of Ferrers
Boards; Proceedings of the American Mathematical Society, Vol. 52, No. 1 (Oct., 1975), pp. 485-492.
[9] Jay R. Goldman, J. T. Joichi, David L. Reiner, Dennis E. White, Rook Theory. II: Boards of
Binomial Type; SIAM Journal on Applied Mathematics, Vol. 31, No. 4 (Dec., 1976), pp. 618-633.
[10] D. C. Foata and M. P. Schutzenberger, On the Rook Polynomials of Ferrers Relations; Col-
loq. Math. Soc. Janos Bolyai, 4, Combinatorial Theory and its Applications, vol. 2, 1970.
[11] John Riordan, An introdoction to Combinatorial Analysis, Moskva 1963 (in Russian)
[12] http : //www.usna.edu/Users/math/wdj/teach/sm342/rook_polynomials.html
[13] http : //mathworld.wolfram.com/notebooks/Combinatorics/RookPolynomial.nb
[14] D. C. Fielder, A Generator of Rook Polynomials; Mathematica J. 9, 371-375, 2004.
Tp ch Ton hc MathVn S 02-2009
Cuc thi gii ton MathVn
Phn A - ton dnh cho Hc sinh
A13. Cho n s t nhin lin tip c xp trn mt vng trn. Bt u nh du cc s c
dng
k(k+1)
2
+ 1, k = 0, 1, 2, ... theo chiu kim ng h. K hiu f(n) l s u tin c nh du
ln th hai.
a. Chng t rng tn ti v hn s n sao cho f(n) = 500501
b. Tm cng thc tng minh ca f(3
k
).
c. M t tt c cc s n sao cho f(n) c xc nh sau n + 1 ln nh du. Tm cng thc
tng minh f(n) i vi cc s n nh vy.
d. Chng t thu hp ca hm s f ln tp cc s nguyn t l, tc l f(p) = f(q) nu p, q l hai
s nguyn t l khc nhau.
PGS Vladimir Lesko, Khoa i s, Hnh hc v Tin hc, i hc S phm Volgograd, LB Nga
A14. Cho hnh lp phng ABCDA
1
B
1
C
1
D
1
c ghp t cc hnh lp phng n v, hai thit
din EKLMN, OPRST song song vi BD c din tch ln lt l 50 v 100. Tnh th tch hnh lp
phng ny.
PGS Vladimir Lesko, Khoa i s, Hnh hc v Tin hc, i hc S phm Volgograd, LB Nga
A15. Cho tam gic ABC vi ng cao AH. Gi M, N ln lt l hnh chiu ca B, C trn
phn gic trong ti nh A. Gi P, Q ln lt l hnh chiu ca B, C trn phn gic ngoi ti nh A
ca tam gic. Chng minh rng hai ng trn (HPQ) v (HMN) trc giao vi nhau. (Hai ng
trn gi l trc giao vi nhau nu gc gi hai tip tuyn ca hai ng trn ti mt im chung ca
chng vung gc vi nhau)
Hong Quc Khnh, Hc sinh Lp 12A10 THPT Chuyn Vnh Phc, Tnh Vnh Phc
A16. Cho tam gic ABC cn ti A. ng trn (O) ni tip tam gic ABC tip xc BC, CA, AB
ln lt ti K, L, M. Gi N l giao im ca OL v KM , Q l giao im ca BN v CA. P l
hnh chiu ca A ln BN. Gi s BP = AP + 2PQ, hy xc nh gi tr ca
AB
BC
.
Nguyn Trn Thi, Th trn Cng Sn, Huyn Sn Ha, Tnh Ph Yn
A17. Cho tam gic ABC din tch S v P l im bt k; gi A

, B

, C

l trung im cc cnh
BC, CA, AB; h
a
, h
b
, h
c
l cc ng cao tng ng. Chng minh rng
PA
2
+PB
2
+PC
2

4
3
S. max
_
PA+PA

h
a
,
PB +PB

h
b
,
PC +PC

h
c
_
Trn Quang Hng, i hc Khoa hc T nhin - HQG H Ni
Tp ch Ton hc MathVn S 02-2009
A18. Cho hai dy v han (a
n
), (b
n
) tha mn a
1
= b
1
v vi n = 0, 1, 2, ... th
b
n+1
= b
n
(1 a
n+1
) + (1 b
n
)a
n+1
Chng minh rng
1
2
l mt phn t ca (a
n
) khi v ch khi n l phn t ca (b
n
)
L Nguyn, Sinh vin Lp TC0662A1, i hc Cn Th
A19. Cho f(x) : R R l hm lin tc, tun hon vi chu k T. Gi F l mt nguyn hm
ca f, chng minh rng vi s thc a bt k th
a+T
_
a
F(x)dx
T
_
0
F(x)dx = a
T
_
0
f(t)dt
Nguyn Vn Vinh, Sinh vin Khoa Ton L, i hc THQG Belarus
A20. Cho cc s thc khng m a, b, c tha mn a +b +c = 6. Chng minh rng
4 a
2
b +b
2
c + 4ca
2
5abc 128
V Quc B Cn - Sinh vin Lp YY0647A1, i hc Cn Th
A21. Cho a, b, c l cc s thc dng, chng minh rng
a. a +b +c +
a b
1 +ab
.
b c
1 +bc
.
c a
1 +ca
3
3

abc
b.
1
a
+
1
b
+
1
c
+
a b
1 +ab
.
b c
1 +bc
.
c a
1 +ca
3
3
_
1
abc
Nguyn nh Thi, Hc sinh Trng THPT Chuyn Lng Vn Chnh, Tnh Ph Yn
A22. Tm gi tr ln nht ca tam thc bc hai f(x) = ax
2
+bx +c ti gi tr x = 2009. Bit rng
hm s nhn gi tr thuc on [0, 1] vi x ln lt bng 0, 1, 1.
Trn Thanh Nam, Sinh vin i hc Bch khoa Tomsk, LB Nga
A23. Cho bng vung n n. Ngi ta t n vung ca bng sao cho khng c hai vung
mu no nm trn cng mt hng hoc cng mt ct. Hi c th t xanh ti a bao nhiu
vung con trong s cc vung cn li sao cho khng c 2 xanh v hai no to thnh 4 nh
ca mt hnh ch nht.
L Hng Qu - Darmstadt, CHLB c
A24. Tm cng thc tnh s cc song nh t tp {1, 2, ..., 2n} vo chnh n sao cho
|(i) i| < n, i = 1, 2, ..., 2n
Nguyn Tun Minh, Sinh vin Lp C nhn Cht lng cao K3, i hc Hu
Tp ch Ton hc MathVn S 02-2009
Phn B - ton dnh cho Sinh vin
B4. Xt A, B M
n
(R) v M =
_
A A
B 0
_
vi rank(B) rank(A). Chng minh rng M tn
ti nghch o suy rng nu v ch nu rank(A) = rank(B) = rank(AB) = rank(BA)
Ghi ch: Ma trn X gi l nghch o suy rng ca ma trn A nu tha mn:
AXA = A, XAX = X, AX = XA
B5. Chng minh vi mi hm s lin tc f : [0, 1] R tho mn bt ng thc sau
1
_
0
f(x)dx.
1
_
0
1 x
n+1
1 x
f(x)dx
1
2
_
n+1

k=1
1
k
_
1
_
0
f
2
(x)dx +
_
n

k=0
k + 1
4k + 2
_
_
_
1
_
0
f(x)dx
_
_
2
Trong n l mt s nguyn dng
Ng Phc Nguyn Ngc, Sinh vin Khoa Xc sut Thng k, i hc THQG Belarus
B6. Tnh tch phn
+
_

_
sinx
x
_
2n+1
dx, n N
Nguyn Vn Vinh, Sinh vin Khoa Ton L, i hc THQG Belarus
Phn C - Cc vn m
1
C1. Xt B l ma trn vung cp n vi rank(B) = m n. t = diag(
1
,
2
, ...,
n
). Chng t
hm C() = trace((B

B)
2
) l li vi = (
1
,
2
, ...,
n
) (R
+
)
n
.
Theo Problems and Solutions, SIAM
C2. Xt phng trnh vi phn u

(x) + u(x)(a(x) u(x)) = 0, 1 < x < 1; v u(1) = u(1) = 0.


Trong a(x) l hm chn lin tc trn [1, 1]. Gi s phng trnh c nghim u(x) > 0 vi mi
x (1, 1), chng t rng nghim ny l hm chn.
Theo Khoa Ton, Trng i hc Khoa hc v Ngh thut McMicken, H Cincinnati, Hoa K
C3. Phi chng xc sut s s nguyn dng c chn ngu nhin (s 2) c c chung l h
bng
1
(s)h
s
. y (s) =

i=1
1
i
s
, s > 1 l k hiu ca hm zeta.
Theo Mehdi Hassani, Khoa Ton, Hc vin Khoa hc C bn, Iran
1
Cc bi ton ca phn ny c chng ti chn ra t nhng vn ca cc khoa Ton, cc trng i hc trn
Th Gii v t cc tp ch Ton hc. Nhng bi ton nh th ny mang tnh cht sinh vin nn chng ti hi vng
cc bn c th cng nhau tham gia gii quyt.
Tp ch Ton hc MathVn S 02-2009
Olympic Hc sinh - Sinh vin
Olympic Sinh Vin Ton Belarus 2009
Bi 1. Cho A
1
, A
2
, ..., A
1066
l cc tp hp con ca tp X hu hn. |X| 10 v |A
i
| >
1
2
|X|
vi mi i = 1, 1066. Chng minh rng trong tp X tn ti 10 phn t sao cho mi tp A
i
cha t
nht mt phn t trong s 10 phn t trn.
Bi 2. Chng ta xem xt mt ton t hai ngi trn mt phng. C nh tam gic XY Z trong
b ba im X, Y, Z c nh du theo chiu ngc chiu kim ng h. i vi bt k hai im
phn bit A, B ca mt phng ta xt ton t A B = C, trong C l nh ca tam gic ABC sao
cho b ba cc im A, B, C v X, Y, Z c cng chiu nh hng v ABC ng dng vi XY Z
(Khi A B th A A = A). Chng minh vi bt k bn im A, B, C, D ca mt phng th ng
thc sau ng
(A B) (C D) = (A C) (B D)
Bi 3. Cho f C

([a, b] ,), 0 [a, b], ng thi f


(n)
(0) = 0 v sup
[a,b]

f
(n)
(x)

n!M
n
, n N,
trong M l hng s. Chng minh f 0.
Bi 4. Tin hnh tung nhiu ln mt ng xu vi xc sut ri vo mt huy hiu (1) v mt s
(0) l nh nhau (1/2). Dy bao gm t cc s 0 v 1 c gi l dy s tha tht nu trong
khng c hai s 1 no nm cnh nhau.
a) Tm xc sut thu c dy tha tht sau n ln tung ng xu.
b) Gi s xc sut ri vo mt c huy hiu l p. K hiu
n
l s cc s 1 c trong mt dy tha
tht ngu nhin c di n. Tnh M[
n
].
Bi 5. Cho E l khng gian vct hu hn chiu trn trng s thc, u, v l hai nh x tuyn
tnh t E vo chnh n. Gi s Ker(u) Ker(v). Chng minh rng tn ti mt nh x tuyn tnh
w : E E sao cho u = w v.
Bi 6. Vi s t nhin c nh m 2 xt nh x
f
m
(x) =

k=0
_
1
km+ 1
+
1
km+ 2
+... +
1
km+m1

x
(k + 1)m
_
Xc nh min xc nh v min gi tr ca hm f
m
.
Tp ch Ton hc MathVn S 02-2009
Olympic Khoa Ton i hc Tng hp Sofia 2009
Dnh cho sinh vin nm I v II
Bi 1. Tp {1, 2, ..., 3n} c phn hoch thnh 3 tp con khng giao nhau, mi tp n phn t.
Chng t rng vi 3 tp con nh vy c th chn t mi tp mt phn t sao cho 1 phn t chn
ra bng tng ca 2 phn t cn li.
Bi 2. a. Chng minh phng trnh x
4
+ 2x
3
4x
2
2x + 1 = 0 c 4 nghim thc phn bit
b. Tnh det
_
_
_
_
1 +x
1
1 1 1
1 1 +x
2
1 1
1 1 1 +x
3
1
1 1 1 1 +x
4
_
_
_
_
, x
1
< x
2
< x
3
< x
4
- cc nghim ca phng
trnh cho.
Bi 3. Cho a thc P(x) bit rng P(2) = 6, P(1) +P(2) +... +P(2n1) = 2nP(2n), n R.
Tnh P(2009).
Bi 4. Tnh lim
n
n

k=1
1
_
n
k
_
Bi 5. Chng minh
_
(n+1)!
e
_
chia ht cho n
Bi 6. Tm tt c cc hm s f : (0, 1) R sao cho 0 < f(x) x v f(x) 2f(x/2)(f(x/2))
2
vi mi x (0, 1)
Bi 7. Vi mi p > 0, liu c tn ti hm f : (0, 1) R sao cho 0 < f(x) x v
f(x) 2f(x/2) (f(x/2))
p
vi mi x (0, 1) hay khng?
Dnh cho sinh vin nm III v IV
Bi 1. tng t bi 1, Bi 2. tng t bi 6 phn trn.
Bi 3. Cho hai hm cng tnh f, g : R R v tha mn f
1
((0, +)) = g
1
((0, +)). Chng
minh rng tn ti c > 0 f(x) = cg(x) vi mi s thc x
Bi 4. Cho f : R R c o hm n cp 3, t L
h
(f, x) = f(x + 2h) 2hf

(x +h) f(x)
h
3
3
f

(x + h). Chng minh L


h
(f, x) = 0 vi mi x, h R khi v ch khi f(x) l a thc vi bc
khng vt qu 4.
Bi 5. Cho nhm hu hn G.
a. Chng minh rng G khng th biu din thnh hp ca 2 nhm con khng tm thng ca n.
b. Gi s G biu din thnh hp ca 3 nhm con khng tm thng ca n. Chng t rng bc
ca G chia ht cho 4
Bi 6. Vi s t nhin n v hm s lin tc f : [0, 1] R tha mn
_
1
0
x
k
f(x)dx = 1 vi
k = 1, 2, ..., n 1. Chng t
_
1
0
(f(x))
2
dx n
2
Tp ch Ton hc MathVn S 02-2009
VMO 2009 thi, li gii v bnh lun
Trn Nam Dng, Trng i hc Khoa hc t nhin, HQG TP H Ch Minh
K thi chn hc sinh gii quc gia mn Ton nm 2009 (VMO 2009) din ra vo ngy 25/2/2009
vi s tham gia ca gn 360 th sinh n t cc tnh thnh. Kt qu 131 th sinh c gii, trong
c 1 gii nht, 22 gii nh, 62 gii ba v 46 gii khuyn khch, t t l 34%. 42 th sinh c im
t 15 tr ln c triu tp tham d k thi chn i tuyn din ra vo na cui thng 4. thi
nm nay c nh gi l d hn nhng cng c nhng bi ton thc s kh khn hn.
T 7 bi xung 5 bi
Sau 2 nm th im thi 7 bi lm trong vng 180 pht, vi kt qu tng ng cc nm 2007,
2008 l 13% v 8% s th sinh ot gii. B gio dc, theo ngh ca cc chuyn gia quyt nh
chn phng n thi gm 5 bi ton lm trong vng 180 pht. C th ni, y l mt thay i
quan trng gip kt qu ca k thi tt hn 2 nm trc.
Mt thay i khc cng nh hng khng nh n kt qu k thi, l vic phn b im cho
cc phn mn c n nh nh sau: Cc bi ton thuc 3 phn mn Gii tch, i s, Hnh hc l
cc phn mn m a s cc th sinh c chun b tt hn, quen hn s c tng im l 14, im
va t gii 3. Trong khi , cc bi ton thuc phn s hc v t hp thuc dng ton l
v kh i vi hc sinh, ch c 6 im. Cch phn b im ny r rng l c li cho s ng cc th
sinh, khin kh nng t gii ca h cao hn.
Tuy nhin, cng cn phi ni rng cch phn b ny cng gy i cht bt li cho cc th sinh
c s trng v s hc v t hp, c bit trong bi cnh k thi VMO nm nay (khi cc bi ton
thuc phn s hc v t hp kh hn hn so vi ba phn cn li). C th s c mt vi th sinh
lm c bi s hc nhng li b bi hnh hoc bi i s. Hoc c th sinh dn sc cho bi t hp
nhng li s sut cc bi d hn. Kt qu l s im t c bi kh khng b c vi s
im b mt bi d.
C th l vn cn c nhng vn cn bn ci, tranh lun, lm th no c c mt thi
tt, phn loi c th sinh v khuyn khch c phong tro nhng nhn chung, thi nm nay
p ng c nhng yu cu c bn nht: S th sinh t gii ng hn; c c cu gii nht,
nh, ba; thi c nhng bi c bn nhng cng c nhng bi kh v hay.
Ba bi c bn
Ba bi ton u, gm bi i s, bi Gii tch v bi Hnh hc l ba bi ton rt c bn, m
theo ngn ng ca cc thy l bi knh biu. Tuy nhin, theo thng tin t ban chm thi th khng
phi th sinh no cng nhn qu biu. Nhiu th sinh b tay vi bi 1. Nhiu th sinh khng lm
c bi hnh hoc b cu b) ca bi ny. Thm ch vi bi 2, bi c coi l d nht ca k thi,
cng c th sinh khng lm c hoc lm c cng tn kh nhiu thi gian. Di y, chng ti
s khng trnh by li gii chi tit m ch bnh lun mt s vn xung quanh bi v li gii.
Cu 1. Gii h phng trnh

1 + 2x
2
+
1

1 + 2y
2
=
2

1 + 2xy

x(1 2x) +

y(1 2x) =
2
9
Tp ch Ton hc MathVn S 02-2009
Bnh lun. R rng ca cc thy ra l mun kim tra kin thc c bn ca hc sinh v
bt ng thc, c th l:
Chng minh rng vi x, y [0, 1] ta c bt ng thc
1

1 +x
2
+
1

1 +y
2

1 +xy
Du bng xy ra khi v ch khi x = y (*).
Bi ny kh quen thuc, xut hin trong nhiu thi cng nh l b ca nhiu bi ton khc
(chng hn thi Nga nm 2000), nu ra thng bt ng thc th qu l nn cc thy thay i
i mt cht, a n vo trong mt h phng trnh. Tt nhin l nu c x = y ri th thay vo
phng trnh th hai, mi vic qu n gin.
Phng php chng minh bt ng thc (*) cng kh a dng. Chng ta im qua cc phng
php .
1. Bnh phng hai v ca bt ng thc, ta c bt ng thc tng ng
1
1 +x
2
+
2

1 +x
2

1 +y
2
+
1
1 +y
2

4
1 +xy
Theo bt ng thc CBS, ta c

1 +x
2
.

1 +y
2
1 +xy
2

1 +x
2
.

1 +y
2

2
1 +xy
Nh vy ta ch cn chng minh
1
1 +x
2
+
1
1 +y
2

2
1 +xy
()
l xong
Nhng (**) qua cc php bin i i s n gin, tng ng vi
(1 xy)(x y)
2
(1 +xy)(1 +x
2
)(1 +y
2
)
0
ng do x, y [0, 1].
2. Ta c th lm khc i mt cht bng cch p dng CBS ngay t u:

1 +x
2
+
1

1 +y
2

2
2

1
1 +x
2
+
1
1 +y
2

4
1 +xy
(theo (**))
T suy ra iu phi chng minh. R rng trong hai cch chng minh trn, ta ch cn iu
kin 1 < xy 1.
3. Gi y c nh, xt hm s f(x) =
2

1 +xy

1

1 +x
2

1 +y
2
trn [0, 1].
Ta c
f

(x) =
x
(1 +x
2
)
3/2

y
(1 +xy)
3/2
=
x
2
(1 +xy)
3
y
2
(1 +x
2
)
3
(1 +x
2
)
3/2
(1 +xy)
3/2
(x(1 +xy)
3/2
+y(1 +x
2
)
3/2
Tp ch Ton hc MathVn S 02-2009
Nh vy du ca f

(x) l du ca
x
2
(1 +xy)
3
y
2
(1 +x
2
)
3
= (x y)(x +y + 3x
2
y x
5
y
2
)
Do x, y thuc [0, 1] nn tha s th hai lun dng, nh th f

(x) i du t m sang dng ti


y, suy ra y l im cc tiu, suy ra f(x) f(y) = 0. Du bng xy ra khi v ch khi x = y.
4. Cn mt cch khc chng minh bt ng thc dng ny, l t x = e
u
, y = e
v
vi
u, v (, 0] a bt ng thc v dng f(u) +f(v) 2f(
u +v
2
)
Trong f(x) =
1

1 +e
2x
.
Tnh o hm bc hai, ta c f(x) =
e
2x
(e
2x
2)
(1 +e
2x
)
5/2
< 0 (do x 0).
Vy hm f(x) lm trn (, 0] v ta c iu cn chng minh.
Cu 2. Cho dy s (x
n
) xc nh bi
x
1
= 1/2, x
n
=

x
2
n1
+ 4x
n1
+x
n1
2
vi mi n 2.
Chng minh rng dy (y
n
) vi y
n
=
n

k=1
1
x
2
k
c gii hn hu hn khi n v tm gii hn .
Bnh lun. y l bi ton d nht ca k thi. Vic chng minh dy (x
n
) tng v khng b chn
trn (tc l c gii hn bng +) l qu n gin. Chng hn c th nh gi:
x
n
=

x
2
n1
+ 4x
n1
+x
n1
2

x
2
n1
+ 2x
n1
+ 1 +x
n1
2
= x
n1
+
1
2
Vic tnh gii hn ca y
n
ch c th thc hin c nu ta tm c cng thc tng minh cho
tng
n

k=1
1
x
2
k
. M iu ny ch c th thc hin thng qua sai phn. Ta bin i tng ng
x
n
=

x
2
n1
+ 4x
n1
+x
n1
2
2x
n
x
n1
=

x
2
n1
+ 4x
n1
4x
2
n
4x
n
x
n1
+x
2
n1
= x
2
n1
+ 4x
n1
x
2
n
x
n
x
n1
= x
n1

1
x
n1

1
x
n
=
1
x
2
n
Sai phn c tm ra, t d dng tm c y
n
= 6
1
x
n
v gii hn cn tm bng 6.
Qu n gin, khng mt cht lt lo, t vic ngh ra li gii n trnh by li gii u n gin.
Tuy nhin trn thc t th by ny cng lm kh cho khng t th sinh. Khng k cc bn khng
gii c, cc bn gii c cng hao tn kh nhiu cng lc bi ny. Nhiu bn c my mc, khi
tm c cng thc cho y
n
ri vn c tip tc i chng minh (y
n
) tng v b chn trn. Nhiu
bn khng bit khi nim gii hn bng !
Bi ton s tr nn kh hn nu bi yu cu tnh lim
n
x
n
n
. Khi s phi vn dng nh l
trung bnh Cesaro hoc dng nh gi cht hn:
Tp ch Ton hc MathVn S 02-2009
x
n
=

x
2
n1
+ 4x
n1
+x
n1
2
<

x
2
n1
+ 4x
n1
+ 4 +x
n1
2
=
x
n1
+ 2 +x
n1
2
= x
n1
+ 1.
x
n
=

x
2
n1
+ 4x
n1
+x
n1
2
>
x
n1
+ 2
2
xn1
+x
n1
2
= x
n1
+ 1
1
x
n1
ton s hay v th v hn nhng i li s khng cn d chu vi i a s cc th sinh. Trong
bi cnh lm 5 bi ton trong vng 180 pht, c l cc thy trnh phng n ny.
Cu 3. Trong mt phng cho hai im A, B (A = B). C l mt im di ng trn mt phng sao
cho ACB = , (0

< < 180

). ng trn tm I ni tip tam gic ABC v tip xc vi cc cnh


AB, BC, CA ln lt ti D, E, F. Cc ng thng AI, BI ct ng thng EF ln lt ti M v N.
a) Chng minh rng on MN c di khng i;
b) Chng minh rng ng trn ngoi tip tam gic DMN lun i qua mt im c nh.
Bnh lun. Bi hnh hc phng ny kh n gin, li gii ch dng kin thc hnh hc lp 9 (t
gic ni tip) v mt cht lng gic. Cu hnh bi ton cng quen thuc v c nhiu tnh cht hay
xung quanh. V d M v N chnh l chn cc ng cao h t B, A xung AI, BI tng ng. Ngoi
ra M, N nm trn cc ng trung bnh ca tam gic ABC (T suy ra MKN = MDN,
trong K l trung im ca AB, suy ra t gic AKDN ni tip, suy ra kt lun phn b) ca
bi ton). Cng c th nhn thy rng ng trng ngoi tip tam gic DMN chnh l ng trn
Euler ca tam gic IAB v do s i qua trung im ca AB. Nhiu th sinh nhn ra iu
ny v kt lun lun. Theo kinh nghim ca chng ti th cc th sinh nn thn trng trong vic s
dng cc kt qu nh vy. Tt nht l nn da vo kt qu chng minh li.
Cui cng, chng ti xin a ra mt s thi Olympic ca Nga lin quan n cu hnh bi ton 3.
1. (Olympic Nga, vng 4, lp 11, 1994) ng trn tm O ni tip tam gic ABC tip xc vi
cc cnh AB, BC v AC ti cc im E, F v D tng ng. Cc ng thng AO v CO ct ng
thng EF ti N v M. Chng minh rng tm ng trn ngoi tip tam gic OMN, im O v
im D cng nm trn mt ng thng.
2. (Olympic Nga, vng 5, lp 9, 1997) ng trn ni tip tam gic ABC tip xc vi cc cnh
AB, BC v CA ti cc im M, N v K tng ng. ng thng i qua nh A v song song
vi NK ct ng thng MN ti im D. ng thng qua A v song song vi MN ct ng
thng NK im E. Chng minh rng ng thng DE cha ng trung bnh ca tam gic ABC.
3. (Olympic Nga, vng 5, lp 10, 1997) ng trn tm O ni tip tam gic ABC tip xc vi
cc cnh AC, AB v BC ti cc im K, M v N tng ng. Trung tuyn BB
1
ca tam gic ct
MN ti im D. Chng minh rng im O nm trn ng thng DK.
4. ng trn tm I ni tip tam gic ABC tip xc vi cc cnh CA, AB tng ng ti E, F;
BI ct EF ti M. Chng minh rng M nm trn ng trung bnh ca tam gic ABC.
Hai bi phn loi th sinh
Trong khi ba bi u tin rt c bn v c phn d th hai bi ton cn li kh kh chu. lm
c hai bi ny, ngoi vic hiu bi, tm c hng i ng, cn cn phi c thi gian. V th,
hai bi ton ny ch dnh cho nhng th sinh tiu dit gn ba bi u trong vng 1,5-2 ting
Tp ch Ton hc MathVn S 02-2009
u tin. Hn na, v n gin b ngoi c th lm nhiu th sinh sa ly.
Cu 4. Cho ba s thc a, b, c tho mn iu kin: vi mi s nguyn dng n, a
n
+b
n
+c
n
l s
nguyn. Chng minh rng tn ti cc s nguyn p, q, r sao cho a, b, c l 3 nghim ca phng trnh
x
3
+px
2
+qx +r = 0.
Bnh lun. y l mt bi ton kh l v kh chu. gii n cn n c kin thc v i s
v s hc. Khng c g cao siu (nh l Viet, cc php bin i i s trn cc biu thc i xng,
tnh cht n gin 2c Z v 2c
2
Z suy ra c Z) nhng li gy kh khn cho cc th sinh.
Chng ta hy bt u bng trng hp 2 chiu ca bi ton: Cho hai s thc a, b tho mn
iu kin: vi mi s nguyn dng n, a
n
+b
n
l s nguyn. Chng minh rng tn ti cc s nguyn
p, q sao cho a, b l 2 nghim ca phng trnh x
2
+px +q = 0.
Theo nh l Viet, r rng iu phi chng minh tng ng vi vic chng minh a+b v ab l
s nguyn. a +b hin nhin nguyn theo iu kin bi. Ngoi ra ta c 2ab = (a +b)
2
(a
2
+b
2
)
l s nguyn.
Ta c th tip tc dng hng ng thc ny suy ra 2a
2
b
2
cng l s nguyn:
2a
2
b
2
= (a
2
+b
2
)
2
(a
4
+b
4
)
n y ta dng b n gin sau:
B . Nu x l s thc sao cho 2x v 2x
2
l cc s nguyn th x l s nguyn.
Chng minh. Ta chng minh bng phn chng. Gi s 2x = k nguyn, nhng x khng nguyn.
Khi k l s nguyn l: k = 2m + 1. Suy ra x = m + 1/2. Nhng khi 2x
2
= 2(m + 1/2)
2
=
2m
2
+ 2m+ 1/2 khng nguyn. Mu thun. Vy iu gi s l sai, tc l x nguyn.
Nh vy, theo b th ab nguyn v ta suy ra iu phi chng minh. T php chng minh ta
cng suy ra kt qu mnh hn: Nu a +b, a
2
+b
2
, a
4
+b
4
l cc s nguyn th a, b l 2 nghim ca
phng trnh x
2
+ px + q = 0 vi p, q l cc s nguyn no (v do a
n
+ b
n
nguyn vi mi n
nguyn dng). iu cng c ngha l ta ch cn dng gi thit ca bi ton n n = 4. V d
a =

2/2, b =

2/2 cho thy k = 4 l gi tr nh nht tho mn iu kin: Nu a, b l cc s thc


tho mn iu kin a
n
+b
n
l s nguyn vi mi n = 1, 2, ..., k th a
n
+b
n
nguyn vi mi n nguyn
dng.
Quay tr li vi li gii ca bi ton VMO 2009. Ta s thy rng k thut khng c g thay i,
tuy c phc tp hn i cht. R rng ta ch cn chng minh a +b +c, ab +bc +ca v abc nguyn.
Theo iu kin bi th a +b +c l s nguyn. Tip theo ta c
2(ab +bc +ca) = (a +b +c)
2
(a
2
+b
2
+c
2
)
l s nguyn.
Tng t nh li gii trn, ta mun chng minh rng 2(ab +bc +ca)
2
cng l s nguyn. T
dng b suy ra ab +bc +ca l s nguyn. iu ny phc tp hn i cht v ng thc tng t
2(a
2
b
2
+b
2
c
2
+c
2
a
2
) = (a
2
+b
2
+c
2
)
2
(a
4
+b
4
+c
4
)
Cha cho ta kt qu mong mun, v
2(ab +bc +ca)
2
= 2(a
2
b
2
+b
2
c
2
+c
2
a
2
) + 4abc(a +b +c) (1)
Tp ch Ton hc MathVn S 02-2009
M ta cha chng minh c abc nguyn. x l iu ny, ta li s dng mt hng ng thc
quen thuc
a
3
+b
3
+c
3
3abc = (a +b +c)(a
2
+b
2
+c
2
ab bc ca) (2)
T y, do a +b +c, a
2
+b
2
+c
2
, a
3
+b
3
+c
3
v 2(ab +bc +ca) l s nguyn nn ta suy ra 6abc
l s nguyn (ta nhn (2) vi 2!). T , nhn (2) vi 3 ta thu c
6(ab +bc +ca)2 = 2(a
2
b
2
+b
2
c
2
+c
2
a
2
) + 12abc(a +b +c)
l s nguyn.
p dng cch chng minh nh b nu trn, ta suy ra ab +bc +ca l s nguyn. T y, thay
vo (2) ta c 3abc l s nguyn.
Tip theo, ta s dng hng ng thc tng t (2)
a
6
+b
6
+c
6
3a
2
b
2
c
2
= (a
2
+b
2
+c
2
)(a
4
+b
4
+c
4
a
2
b
2
b
2
c
2
c
2
a
2
)
vi ch 2(a
2
b
2
+b
2
c
2
+c
2
a
2
) l s nguyn ta suy ra 6a
2
b
2
c
2
l s nguyn. T 6abc v 6a
2
b
2
c
2
l s
nguyn, bng cch chng minh hon ton tng t ta suy ra abc l s nguyn. Bi ton c gii
quyt hon ton.
Li gii trn cho thy rng chng ta ch s dng gi thit a
n
+b
n
+c
n
cho n n = 6 (trong
khng s dng gi thit vi n = 5!). Lin quan n vn ny, chng ti xut cc c gi suy
ngh n cc vn sau:
1. Chng minh b tng qut: Cho k > 1 l mt s nguyn phi chnh phng (khng c c
chnh phng), (m, k) = 1. Khi nu x l mt s thc sao cho kx v mkx
2
l s nguyn th x l
s nguyn.
2. Hy tm v d v b ba s thc a, b, c tho mn iu kin a
n
+ b
n
+ c
n
nguyn vi mi
n = 1, 2, 3, 4, 5 nhng a
6
+b
6
+c
6
khng nguyn.
3. (Gi thuyt) C phi chng nu a
n
1
+ a
n
2
+ ... + a
n
k
l s nguyn vi mi n nguyn dng th
a
1
, a
2
, ..., a
k
l nghim ca mt a thc n khi bc k vi h s nguyn? V cc hng s 4 (i vi
k = 2) v 6 (i vi k = 3) s bng bao nhiu trong trng hp tng qut?
Cu 5. Cho s nguyn dng n. K hiu T l tp hp gm 2n s nguyn dng u tin. Hi c
bao nhiu tp con S ca T c tnh cht: trong S khng tn ti cc phn t a, b m |a b| {1, n}
(Lu : Tp rng c coi l tp con c tnh cht nu trn).
Bnh lun. Bi ton c v kh quen thuc. Chng hn mt bi ton ni ting Tm s tt c
cc xu nh phn di n sao cho khng c 2 bt 1 ng k nhau c th pht biu mt cch tng
ng l Tm s tt c cc tp con S thuc {1, 2, ..., n} sao cho trong S khng tn ti hai phn t
m |a b| = 1.. Kt qu bi ton ny l chng ta s c dy s Fibonacci.
Tng t, mt bi ton khc c v b ngoi cn ging hn na, l thi ca Thu S nm
2006: Cho s nguyn dng n. Tm s tt c cc tp con A {1, 2, ...., 2n} sao cho khng tn ti
x, y A vi x +y = 2n + 1.
Tuy nhin, ch l ci ging b ngoi. Bn cht bn trong th bi VMO 2009 kh hn hn.
Vi bi xu nh phn di n, ta c th d dng lp c cng thc truy hi x
n
= x
n1
+ x
n2
bng cch l lun nh sau: Xt 1 xu nh phn di n tho mn bi. Nu xu ny bt u
bng 0 th b s 0 ny i, ta c 1 xu nh phn di n 1 tho bi v ngc li, nu c
Tp ch Ton hc MathVn S 02-2009
1 xu nh phn di n 1 tho bi th thm s 0 vo pha u, ta c 1 xu nh phn
di n tho bi bt u bi 0. T s xu loi ny bng x
n1
. Nu xu ny bt u bng 1
th ch s tip theo phi l 0. B i hai ch s u tin, ta c 1 xu nh phn di n 2 tho
bi. T s xu nh phn di n tho bi bt u bng 1 bng x
n2
. Vy x
n
= x
n1
+x
n2
.
Bi Thu S 2006 c li gii trc quan kh n gin nh sau: Ta xp cch s t 1 n 2n thnh
2 hng, n ct
1 2 3
. . .
n
2n 2n-1 2n-2
. . .
n + 1
Khi bi ton tng ng vi vic tm s cch chn ra mt s , sao cho hai cng ct khng
ng thi c chn. Nh vy, ti mi ct ta ch c 3 la chn: hoc khng chn c hai, hoc chn
s hng trn, hoc chn s hng di. Suy ra p s bi ton l 3n.
T li gii trn, d dng nhn thy rng Thu S 2006 tng ng vi bi Tm s cc tp con
S thuc T sao cho khng tn ti a, b thuc S vi |a b| = n. Lc ny ta ch cn i bng thnh
1 2 3
. . .
n
n+1 n+2 n+3
. . .
2n
Li gii cc bi ton trn tuy kh xa vi li gii VMO 2009 (thm ch bi Fibonacci c th dn
n sa ly) nhng gi cho chng ta mt s tng: xy dng cng thc truy hi, a v m hnh
trc quan bng.
Sau y l li gii chi tit:
Ta t cc s thuc {1, 2, ..., 2n} vo bng 2 n nh sau
1 2
. . .
n-1 n
n+1 n+2
. . .
2n-1 2n
Bi ton ca chng ta tng ng vi vic m s cch chn mt s s (c th l khng s no)
sao cho:
i. Hai s k nhau trong bng khng ng thi c chn
ii. n v n + 1 khng ng thi c chn.
Gi S(n) l p s ca bi ton. K hiu A(n), B(n), C(n) l s cc cch chn mt s s (c th
khng s no) t cc bng hnh bn di sao cho khng c hai s no cnh nhau c chn.
Khi d thy rng S(n) = A(n) C(n 2). V ta c cc h thc sau:
A(n) = A(n 1) + 2B(n 1) (1)
Tp ch Ton hc MathVn S 02-2009
(ta chia cc cch chn ca A(n) thnh ba trng hp: c 1 v n + 1 u khng c chn; 1 c
chn; n + 1 c chn)
B(n) = A(n 1) +B(n 1) (2)
(l lun tng t)
C(n) = B(n 1) +B(n 2) +C(n 2) (3)
(1 khng c chn; 1 c chn v n + 2 khng c chn; 1 c chn v n + 2 c chn)
T cc h thc ny ta c th tnh c A(n), B(n), C(n) v S(n). V d, ta c th d dng suy
ra
A(n) = 2A(n 1) +A(n 2), A(0) = 1, A(1) = 3, A(2) = 7. (4)
v t
A(n) =
(1 +

2)
n+1
+ (1

2)
n+1
2
,
C(n) c tnh t h thc
C(n) = C(n 2) +A(n 1) (5)
(B(n 1) +B(n 2) = A(n 2) +B(n 2) +B(n 2) = A(n 1))
C(n) = C(n 2) +
A(n)A(n2)
2
C(n)
A(n)
2
= C(n 2)
A(n2)
2
Ta ch rng C(1) = 1, C(2) = 4, C(3) = 8. T h thc cui cng v
C(1) A(1)/2 = 1/2, C(2) A(2)/2 = 1/2, C(3) A(3)/2 = 1/2
Ta suy ra rng
C(n) A(n)/2 = (1)
n
/2
Nh th
C(n) =
A(n) + (1)
n
2
V cui cng
S(n) = A(n) C(n 2) =
(3 +

2)(1 +

2)
n
+ (3

2)(1

2)
n
2(1)
n
4
.
Ghi ch
1. T (1), (2), (3) c th suy ra rng S(n) = S(n1)+3S(n2)+S(n3). Chng ti khng r l c
th tm c cch chng minh trc tip h thc ny m khng thng qua cc dy s ph c khng.
2. Bi ton s d chu hn nu yu cu khng tn ti a, b sao cho |a b| {1, n} c i thnh
khng tn ti a, b sao cho |a b| = 1 hoc a +b = 2n + 1. Khi p s s chnh l A(n).
Tp ch Ton hc MathVn S 02-2009
Gc lp trnh tnh ton
th trong Mathematica
th l mt phn quan trng trong cc chng trnh ton. Mathematica l phn mm h tr
kh tt v mt ny. Mathematica c tp hp cc cu lnh cho php chng ta xy dng phn ln
th ca cc hm ton hc. C th s dng Mathematica v th ca hm mt bin, v mt lc
nhiu th, v nhng mt phc tp trong khng gian ba chiu. Trong th, chng ta c th km
theo vn bn, m thanh, v c th to ra mt lot hnh nh (k c hnh nh ng). Ngoi ra, khi
thay i tham s trong nhng cu lnh, ta c th thay i mu nn, khung.
u tin chng ta xt kh nng ca h thng Mathematica xy dng th ca hm mt
bin. v th hm mt bin chng ta s dng hm Plot, hm ny c th c s dng nh
sau:
- Plot[f, {x, xmin, xmax}] - dng xy dng th hm trong khong t xmin n xmax.
- Plot[{f1, f2, ...}, {x, xmin, xmax}] - dng xy dng dy th ca cc hm f1, f2,... trong
khong t xmin n xmax.
- Plot[Evaluate[f], {x, xmin, xmax}] - u tin n chuyn f sang dng s, v sau t n v
th trong khong cho.
- Plot[Evaluate[y[x]/.solution], {x, xmin, xmax}]- n xy dng th ca nghim s phng
trnh vi phn nhn c khi s dng lnh NDSolve.
- Plot[Evaluate[Table[f, ...]], {x, xmin, xmax}] to ra danh sch hm s, sau xy dng
th ca chng trong khong cho.
Ta a ra mt vi v d minh ha:
1. Xy dng th hm Sin[x] trong khong (, )
Plot[Sin[x], {x, , }]
2. By gi, yu cu v th ng trn c cho di dng tham s

x = sint
y = cos t
trong
t (, ).
Tp ch Ton hc MathVn S 02-2009
Nu chng ta dng lnh Plot[{Sin[t], Cos[t]}, {t, , }] ta s nhn c th nh trn
iu ny khng ng yu cu ca chng ta. y ta phi dng mt cu lnh khc l
ParametricPlot:
- ParametricPlot[{x, y}, {t, tmin, tmax}] - xy dng th ca hm theo tham s t, trong t
nhn gi tr t tmin n tmax.
- ParametricPlot[{{x1, y1}, {x2, y2}, ...}, {t, tmin, tmax}] xy dng ng thi th ca nhiu
hm cho bi tham s.
i vi th d trn
ParametricPlot[{Sin[t], Cos[t]}, {t, , }]
Ngoi cc i s bt buc, hm xy dng th c mt s lng ln cc ty chn, dng lm
cho th ni bt hn, d nhn hn. Nhng la chn c cho vi cu trc chung name value. Gi
tr ca chng c th l s, danh sch, gi tr logic True hoc False, v nhng t c bit: Automatic
(s dng t ng ty chn), None (cc ty chn khng c s dng), All (ty chn c s dng
trong mi trng hp). By gi chng ta lm quen vi mt vi ty chn (i vi nhng ty chn c
du sao trn u c th s dng cho th 3 chiu):
- AspectRatio* - a ra t l ca chiu cao i vi chiu rng (mc nh l 1/GoldenRatio);
- Axes* - xc nh cn c v trc ta hay khng (mc nh Automatic);
- AxesLabel* - cn a vo ch i vi cc trc ta hay khng (mc nh None);
- AxesOrigin - ch ra ni t gc ta (mc nh Automatic);
Tp ch Ton hc MathVn S 02-2009
- AxesStile* - ch ra loi (style) no xy dng trc ca th (mc nh Automatic);
- Background* - ch ra mu nn ca th (mc nh Automatic);
- ColorOutput* - ch ra mu sc ca th (mc nh Automatic);
- DefaultFont* - ch ra font vit ln th (mc nh $DefaultFont);
- Frame ch ra cn c v khung quanh th hay khng (mc nh False);
- FrameLabel ch ra c t trn bin ca khung cc ghi ch hay nhn hiu (mc nh None);
- FrameTicks ch ra mt phn b ca cc nhn hiu trn bin ca khung (mc nh
Automatic);
- GridLines ch ra c cn xy dng cc ng li trn th hay khng (mc nh None);
- MaxBend a ra gi tr ln nht ca ng cong khi v th hm s (mc nh 10);
- PlotDivision ch ra s on ln nht khi v th ca ng cong trn (mc nh 20);
- PlotLabel* cho bit c a vo nhn ca th hay khng (mc nh - None);
- PlotRange* ch ra cc loi im nm trong th (mc nh - Automatic);
- Ticks* ch ra c nh du cc gi tr trn cc trc hay khng (mc nh - Automatic).
xc nh gi tr cc ty chn ta c th dng hm sau Options[Name].
Khi xy dng cc mt 3 chiu ta s dng cc hm c bn sau:
- Plot3D[f, {x, xmin, xmax}, {y, ymin, ymax}] xy dng th 3 chiu ca hm s f(x, y), trong
min xc nh cho;
- ParametricPlot[{x, y, z}, {t, tmin, tmax}] xy dng th c cho di dng tham s

x = x(t)
y = y(t)
z = z(t)
trong t nhn gi tr t tmin n tmax;
- ParametricPlot[{x, y, z}, {t, tmin, tmax}, {u, umin, umax}] xy dng th 3 chiu c cho
di dng tham s (2 tham s), trong t v u c cho trong cc khong tng ng (tmin,tmax)
v (umin, umax).
- Thng thng, theo mc nh khi v cc mt 3 chiu th th cha ng trong mt hp,
nu chng ta khng mun ty chn ny chng ta c th vit Boxed False.
- Khi xy dng th, thnh thong, xy ra s bin i dng, tham s v cc ty chn. Trong
trng hp ny, tt hn chng ta c th s dng cc hm sau:
- Show[plot, option value] xy dng th vi ty chn cho trc;
- Show[plot1, plot2,...] xy dng mt s th v kt hp chng li vi nhau.
Tp ch Ton hc MathVn S 02-2009
Chng ta xt thm mt v d:
g = ParametricPlot3D[{uSin[t], uCos[t], t/3}, {t, 0, 2}, {u, 1.3, 1.3},
AxesLabel {X, Y, Z}, PlotLabel Vidu]
Show[g, Boxed False, Axes None, ViewPoint {0, 1, 1}]
Trong cc phin bn Mathematica gn y, kh nng x l th l rt rng. Trong bi vit ny
ch im qua s lc cc hm cng nh nhng ty chn.
Tp ch Ton hc MathVn S 02-2009
Tin tc Ton hc
A - Tin Th Gii
Gii thng Abel 2009
Gii thng Abel 2009 c trao cho Mikhail Leonidovich Gromov, gio s thng trc, Vin
Hautes tudes Scientifiques (Php) v gio s Vin Ton Courant, H New York, ng c vinh
danh v nhng ng gp mang tnh cch mng cho hnh hc. Gii thng ny tr gi 6 triu Kroner
Thy in (tng ng vi gn 950.000 USD).
Leonidovich Mikhail Gromov sinh ngy 23-12-1943 ti Boksitogorsk, Lin X c. ng nhn
bng Thc s nm 1965, bo v Tin s nm 1969 v Tin s Khoa hc nm 1973 ti i hc
Leningrad, ni ng lm tr ging t 1967 n 1974. K t 1982, Gromov c b nhim vo v
tr Gio s thng trc ti Vin Hautes tudes Scientifiques, Bures-sur-Yvette, Php. IHS l mt
vin nghin cu chuyn su v ton hc, vt l l thuyt v cc lnh vc lin quan n khoa hc.
ng nhp quc tch Php t nm 1992. Mikhail Gromov L. v hin nay cng l cng l gio s ti
Vin Ton Courant, i hc New York, Hoa K.
Tn tui ca Gromov mi mi gn lin vi nhng kt qu su sc v nhng khi nim quan
trng i vi hnh hc Riemann, Symplectic Geometry, String Theory v l thuyt nhm. Hi ng
ca gii thng nhn xt rng: "Mikhail Gromov lun lun theo ui cc vn mi v khng
ngng suy ngh v nhng tng mi cho cc li gii ca nhng vn c. ng cho chng ta
thy nhng ng trnh su sc trong sut s nghip ca mnh v nhng s sng to ng ch . Cc
Tp ch Ton hc MathVn S 02-2009
cng trnh ca Gromov s tip tc l ngun cm hng cho nhiu khm ph ton hc trong tng
lai". Mikhail L. Gromov nhn c nhiu gii thng quc t khc nh Kyoto Prize in Basic
Sciences (2002), Balzan Prize (1999), Leroy P. Steele Prize for Seminal Contribution to Research
(1997), Lobatchewski Medal (1997) v Wolf Prize (1993). ng l thnh vin ca Vin Hn lm khoa
hc quc gia Hoa K, Vin Hn Lm Khoa hc v Ngh Thut Hoa K v Vin hn lm khoa hc
Php. ng l thnh vin th 3 ca Vin Ton Courant nhn c gii Abel (Peter Lax, nm 2005
v Srinivasa S.R. Varadhan nm 2007).
"Cc cng trnh ca Leonidovich Mikhail Gromov nh hng to ln n ngnh hnh hoc v t
c nhiu ng dng rng ri trong gii tch v i s. Hi Ton hc Hoa k nhit thnh gi li
chc mng n gio s Mikhail Gromov nhn dp ng c nhn gii thng Abel 2009. Khng ai
c th tng c c iu g cn xng ng hn" - George Andrews, Ch tch AMS.
GS Gromov c nh vua Thy in Harald trao gii thng Abel vo ngy 19, thng 5, 2009
ti Oslo.
Thng tin c th xem thm ti: http://www.abelprisen.no/no/nyheter/nyhet.html?id=180
Hi ngh ca SIAM v iu khin v ng dng 2009
Nm nay Cng ng Ton ng dng v Cng nghip th gii (SIAM) s t chc hi ngh v iu
khin v ng dng (SIAM Conference on Control and Its Applications) t ngy 06 n 08 thng
07, 2009 ti khch sn Sheraton Denver Downtown, Denver, bang Colorado, Hoa K.
Lnh vc ca l thuyt iu khin lin quan n hng lot cc vn v hng khng dn dng,
hng khng v tr, cc h thng t ng ha v k thut cao v c xem nh l nn tng cho nhng
cng ngh tin tin ang pht trin mnh hin nay t cng ngh nano n cng ngh iu chnh
t bo. L thuyt ny ng dng rng ri cho cc ngnh vt l, sinh hc, my tnh v khoa hc x hi.
Hi ngh s trnh by hng lot cc vn v l iu khin v h thng v nhng ng dng
lin quan n real-time optimization, data assimilation, cellular v iu tit sinh hc, k thut iu
khin v ton ti chnh, iu khin y-sinh, iu khin h thng thng minh, iu khin dng chy
v iu khin lng t... Hi ngh nm 2009 l s tip ni lot hi ngh bt u t t chc u tin
ti San Francisco nm 1989.
Thng tin thm c th xem ti: http://www.siam.org/meetings/ct09/
Tp ch Ton hc MathVn S 02-2009
Ch tch ca cc hip hi Ton hc Chu u nhm hp ti Warsaw
Trong hai ngy 0910 thng 05, 2009 ti Banach Centre, Th Warsaw, Ba Lan, din cuc
gp g gia ch tch cc hip hi Ton hc cc quc gia trn ton Chu u do Hip hi Ton hc
Chu u ch tr (EMS).
c bit y l ln gp g th 2 m EMS t chc thnh cng. Ln th nht din ra ti
Luminy, Php thng 04 nm ngoi. EMS d nh s t chc thng nin s kin ny tng cng
s hu ngh hp tc ca ton th cc nh Ton hc trn ton chu u.
Tin tc v hi ngh ny c th xem thm ti: http://www.euro-math-soc.eu/node/263
Cuc thi Olympic Ton hc sinh vin Th gii ln th 16
Nm nay cuc thi Olympic Ton hc sinh vin Th gii (IMC) c t chc ti th Bu-
dapest, Hungary t ngy 25-30 thng 07 di s phi hp gia i hc London (Anh Quc) v
i hc E otv os Lornd, i hc Cng ngh v Kinh t Budapest. Cuc thi c s tham gia ca cc
sinh vin t nm 1 n nm 4 cc trng i hc trn ton th gii, c chia lm hai vng, mi
vng c thi gian lm bi l 5 ting ng h. Cc bi ton c ngh thuc lnh vc i s, Gii
tch (thc v phc), Hnh hc v T hp. Cc th sinh phi s dng ting anh trong bi thi ca mnh.
c bit trong sut cc k thi IMC c t chc t nm 1994 thu ht sinh vin ca hn
150 trng i hc t 40 quc gia trn ton th gii tham d.
Thng tin c cp nht t: http://www.imc-math.org/
Tp ch Ton hc MathVn S 02-2009
B - Tin trong nc
Trng h Ton hc cho sinh vin
Nm nay vin ton tip tc m trng h to iu kin cho nhiu bn sinh vin khoa Ton trn
ton quc tham gia trao i v giao lu. Trng h l hot ng chnh ca n Nng cao nng
lc nghin cu Ton hc c Qu Pht trin Khoa hc v Cng ngh quc gia ph duyt v ti
tr kinh ph. Mc ch ca n l h tr sinh vin gii ca cc trng i hc pht huy c kh
nng hc tp ca mnh, tp dt nghin cu trong qu trnh hc i hc. Qua s tng s sinh
vin tt nghip i hc c kh nng nghin cu Ton.
Trng h Ton hc cho sinh vin nm 2009 s c t chc ti Vin Ton hc, l bc tip
ni ca Trng h 2008. Ni dung ca Trng h bao gm 6 lot bi ging v cc hng khc nhau
vo cc bui sng v cc bo co, tho lun vo cc bui chiu. Vin Ton hc c th ti tr cho
mt s sinh vin xut sc tham d Trng h, bao gm chi ph i li bng tu (v nm cng), ti
H Ni (k tc x ca Trng HSP H Ni) v h tr mt phn sinh hot ph.
Thng tin tham kho t Vin Ton hc Vit Nam: http://www.math.ac.vn/
i tuyn Vit Nam tham d IMO 2009 ti c
Kt thc hai t thi quc gia v TST cui cng chng ta cng c 6 i din thuc nhiu tnh
thnh khc nhau c mt ti BREMEN, c vo h ny:
1) H Khng Duy (HKHTN - HQG HN)
2) Nguyn Xun Cng (Hi Dng)
3) Phm Hy Hiu (PTNK - HQG Tp HCM)
4) T c Thnh (Ph Th)
5) Nguyn Hong Hi (Vnh Phc)
6) Phm c Hng (Hi Phng)
Thi gii Ton cp quc gia qua Internet
Ngy 12/5 va qua, i tuyn Ton lp 5 v lp 9 ca 63 tnh, thnh ph tham d cuc thi
"Gii Ton qua Internet" (ViOlympic Ton hc) dnh cho hc sinh ph thng do B Gio dc &
o to v Tp on FPT ln u t chc.
Mi S Gio dc & o to thnh lp 2 i tuyn (lp 5 v lp 9), mi i t nht 10 thi sinh.
Nhng em ny phi c tuyn chn trong s cc th sinh tham d y cc k thi cp trng,
Phng Gio dc v S Gio dc & o to.
y ang l mt hot ng thu ht c s ch ca ng o hc sinh tham gia.
Xem thm ti: http://www.violympic.vn/
Tp ch Ton hc MathVn S 02-2009
Ch tch v Tng Th k Lin on Ton hc Th gii thuyt trnh ti HQGHN
Ngy 4/3/2009 GS. Laszlo Lovasz - Ch tch v GS. Martin Groetschel - Tng Th k Lin on
Ton hc Th gii c bui gp g, thuyt trnh vi cn b, ging vin cng sinh vin ngnh ton
hc ca mt s trng i hc ng trn a bn H Ni.
Bui gp g do Trng HKHTN HQGHN phi hp cng vi Hi Ton hc Vit Nam,
Trng H S phm H Ni v Vin Ton hc Vit Nam t chc.
GS.TS Nguyn Hu c Ph Gim c HQGHN ti d v chc mng s hin din ca
hai gio s ti HQGHN.
Ti bui giao lu, ngi Ch tch Laszlo Lovasz trnh by bo co chuyn lm th no
hon thin mt th (How to draw graphs?); GS. Martin Groetschel trnh by bo co v nhng
ng dng ca ton hc trong cuc sng hin i hin nay ("Mathematics in everyday life"). Cng
trong khn kh ca bui gp g, hai ng c nhng trao i trc tip vi sinh vin v mt s vn
thi s v vai tr ca ton hc ngy nay. y thc s l nhng thng tin hu ch v hp dn vi
nhng ngi yu ton hc.
Thng tin c cp nht t: http://www.hus.edu.vn/News
Tp ch Ton hc MathVn S 02-2009
Olympic Ton Sinh vin 2009
Nhm gp phn nng cao cht lng dy v hc ton, thc y phong tro hc tp ca sinh
vin, ng thi gp phn pht hin, bi dng cc sinh vin gii ton cc trng i hc v cao
ng, k thi Olympic Ton Sinh Vin c t chc ti Qung Bnh t ngy 15 n 20 thng 04
nm 2009 di s phi hp gia Hi Ton hc Vit Nam, B Gio dc v o to, i hc Qung
Bnh, Lin hip cc hi khoa hc v k k thut Vit Nam, Hi Sinh vin Vit Nam.
Nm nay c s tham gia ca gn 650 sinh vin ca 69 trng i v Cao ng trong c nc
d thi hai mn l Gii tch v i s. Kt qu c hai bn Nguyn Trng Ngha, Trng i hc
Bch khoa Thnh ph H Ch Minh v bn Nguyn Trn Thun, Trng i hc Vinh cng t
tng s im tuyt i l 30.
thi v danh sch sinh vin t gii c th xem : http://www.vms.org.vn/
Hi ngh i s - Hnh hc - T p, Hu 2009
Hi ngh i s-Hnh hc-Tp c t chc hai nm mt ln. Mc ch ca Hi ngh l to
iu kin cho cc nh nghin cu, ging vin ang cng tc ti cc vin nghin cu, cc trng i
hc v cao ng trong c nc trao i cc kt qu nghin cu t c trong thi gian gn y v
cc lnh vc i s, Hnh hc v Tp.
Xem thm ti: https://sites.google.com/site/hoitoanhochue/dahito
Mi sut hc bng du hc ton phn ti Lin bang Nga
Vo ngy 10-5, ti hai a im l Trng THPT Nguyn Bnh Khim (H Ni) v hi trng
L Vn Thim ca H Quc gia H Ni ti 19 L Thnh Tng din cuc thi dnh hc bng du
hc ti Nga.
Mi sut hc bng ton phn du hc ti Lin bang Nga c trao cho mi th sinh t kt
qu cao nht trong cuc thi Olympic ton hc dnh cho hc sinh lp 12 do Khoa Quc t (H Quc
gia H Ni) phi hp vi Hip hi cc trng i hc ti Lin bang Nga t chc.
Mi sut hc bng ton phn du hc ti Lin bang Nga bao gm hc ph cc giai on hc d
b ting, chng trnh o to H, nh v sinh hot ph ti mt trong s 22 trng H thuc.
Hip hi cc trng H Lin bang Nga.

You might also like